Paramedic CP1

Ace your homework & exams now with Quizwiz!

What is the biggest preventable injury in the geriatric population?

Falls

Denial, anger, bargaining, depression, and acceptance are the correct order for the stages of death.

False

As the thoracic cavity begins to expand, the intrathoracic pressure:

Is less than atmospheric pressure

Which of the following is an example of how EMS providers can be involved in community injury prevention?

Participating in a high school drunk driving crash scenario

All of the following are signs of inadequate breathing in an infant EXCEPT: cyanosis. altered mental status. a respiratory rate of 32 breaths per minute. use of accessory muscles.

a respiratory rate of 32 breaths per minute.

homeostasis

the natural tendency of the body to maintain a steady and normal internal environment

simple diffusion

the passive movement of molecules through a membrane from an area of greater concentration to an area of lesser concentration

hematocrit

the percentage of the blood occupied by erythrocytes

pathology

the study of disease and its causes

epidemiology

the study of factors that influence the frequency, distribution, and causes of injury, disease, and other health-related events in a population

metabolism

the total changes that take place during physiologic processes; the body's breaking down of chemicals into different chemicals

Which of the following best describes the technique of evaluating plantar reflexes?

Stroke the lateral aspect of the sole of the foot from the heel to ball, curving medially

A five-year-old child is introduced to you as Robert Smith. The proper way for you to address him is: "Hello, Mr. Smith." "Hi, Robert." "Hi, Bobby." "Hey, little guy!"

"Hi, Robert."

You are tasked to establish an IV in a dehydrated 5-year-old child who has been vomiting. You are also going to administer an antiemetic. What is the best way to approach the child?

"I'm going to give you some medicine to help you stop throwing up, but the medicine has to go through a tube and into your arm. So it's going to feel like you're getting a shot, but it will just take a second, and then the medicine will make you feel better."

shock signs/symptoms: hypovolemic

"classic" signs, altered mental status, progression of anxiety to lethargy to combativeness or unresponsiveness, pale/cool/clammy skin, normal blood pressure during compensation that then begins to fall, pulse progresses from normal to rapid to slow to disappearing

34) Which of the following processes is used when glucose enters a body cell? A) Diffusion B) Facilitated diffusion C) Active transport D) Hydrostatic pressure Answer: B

) Facilitated diffusion

The normal range of hemoglobin (Hb) levels in the adult male patient would be closest to: 9.9-14.5 g/dL. 14-17.4 g/dL. 9.9-14.5 mg/dL. 14-17.4 mg/dL.

14-17.4 g/dL.

Of the normal tidal volume for the average 70 kg adult, what amount of air (in mL) is NOT available for gas exchange?

150

The automatic transport ventilator is contraindicated for all of the following intubated patients EXCEPT:

17-year-old gunshot victim

The practice of notifying a receiving hospital of an ambulance's impending arrival began in the:

1970s

Key events in the historical development of EMS include all of the following, EXCEPT the:

1975 Cater-Benson Reform Act

38) When cells are moderately differentiated with some structural similarity to the tissue of origin and the prognosis is fair, this is what grade of cancer cell? A) 2 B) X C) 1 D) 3 Answer: A

2

An elderly male is complaining of shortness of breath. You discover pitting edema that is 1/2 to 1 inch deep. This is _____ pitting edema.

2+

The tip of the endotracheal tube for the pediatric patient should be inserted _________ cm below the vocal cords.

2-3

A major advance in the development of EMS as a true health care profession was the:

2009 publication National EMS Services Education Standards

When a bag-valve ventilation device is used without supplemental oxygen, it delivers an oxygen concentration of _________ percent.

21

During a respiratory assessment, the absence of breath sounds may indicate A) A pneumothorax B) A pulmonary edema C) Flail chest D) Bronchitis

A) A pneumothorax

Which of the following instances might digital intubation NOT be indicated? A) Airway is blocked by a foreign body B) Patient has copious secretions C) Your equipment fails D) Patient position does not allow for visualization

A) Airway is blocked by a foreign body

48) You are presented with a patient displaying urticaria, dyspnea, hypotension, nausea, vomiting, and dizziness. This patient is MOST likely suffering from: A) Anaphylaxis B) Hypovolemia C) Sepsis D) Cardiogenic failure Answer: A

A) Anaphylaxis

46) Which of the following adaptive mechanisms for cellular injury is LEAST likely to result in the proliferation of malignant cells? A) Atrophy B) Dysplasia C) Metaplasia D) Hyperplasia Answer: A

A) Atrophy

What does the "B" in BURP stand for? A) Backward B) Below C) Bloody D) Behind

A) Backward

When a bag-valve ventilation device is used with supplemental oxygen, it delivers an oxygen concentration of ___ percent. A) 40 B) 90 C) 60 D) 21

B) 90

Applying posteriorly directed pressure on the cricoid cartilage to facilitate endotracheal intubation is also referred to as A) Digital intubation B) Cricoid pressure C) Retrograde intubation D) Cricothyrotomy

B) Cricoid pressure

A properly placed ___ effectively isolates the trachea for ventilation and protection from aspiration. A) Dual lumen airway B) Endotracheal tube C) Laryngeal mask airway D) King LTD airway

B) Endotracheal tube

When swallowing occurs, the structure that occludes the tracheal opening to prevent aspiration of food and liquid is the A) Cricoid cartilage B) Epiglottis C) Pyriform fossa D) Uvula

B) Epiglottis

Which of the following is an objective technique to verify proper endotracheal tube placement? A) Direct visualization B) Esophageal detector device C) Tube misting D) Auscultation

B) Esophageal detector device

During spontaneous breathing, which of the following reflexes prevents overinflation of the lungs? A) Cheyne-Stokes B) Hering-Breuer C) Apneustic D) Cushing's

B) Hering-Breuer

Which of the following terms can be defined as the activities of the body that allow the maintenance of physiologic stability? A) Sympathetic tone B) Homeostasis C) Inflammation D) General adaptation syndrome

B) Homeostasis

When using a colorimetric end-tidal carbon dioxide detector, the absence of carbon dioxide in exhaled air after six breaths indicates the endotracheal tube has been placed A) In the right mainstem bronchus B) In the esophagus C) In the left mainstem bronchus D) In the trachea

B) In the esophagus

Which of the following is a response to histamine release? A) Release of leukotrienes B) Increase in blood flow at the site of injury C) Decrease in vascular wall permeability D) Suppression of lysosomal enzymes Answer: B

B) Increase in blood flow at the site of injury

Which of the following best describes a comprehensive network of personnel, equipment, and resources established to deliver aid and emergency medical care to the community?

EMS system

Paramedics carry out their tasks in the prehospital setting as designated agents of the:

EMS system medical director

bystanders.

EMS providers must routinely communicate effectively​ with: A. media. B. attorneys. C. auditors. D. bystanders.

To practice, paramedics must be approved not only by a state or provincial agency but by the:

EMS system medical director

Which of the following is an advantage of the two-person method of bag-valave-mask ventilation?

Easier to maintain a seal between the patient's face and the mask

What is the highest level of prehospital care provider?

Emergency Medical Responder (EMR)

The ________, the public's first contact with the EMS system, plays a crucial role in every EMS response.

Emergency medical dispatcher

Which of the following is NOT typically a role of the medical director in an EMS system?

Employee scheduling

A properly placed ________ effectively isolates the trachea for ventilation and protection from aspiration

Endotracheal tube

A(n) _________ may be used to facilitate nasotracheal intubation

Endotrol

Palpation and inspection of the point of maximal impulse can reveal certain conditions, such as:

Enlarged right ventricle

When swallowing occurs, the structure that occludes the tracheal opening to prevent aspiration of food and liquids is the:

Epiglottis

With which of the following conditions should you NOT attempt endotracheal intubation of the patient unless airway failure is imminent?

Epiglottitis

At the end of exhalation, the intrathoracic pressure is:

Equal to atmospheric pressure

The rules, standards, and expected actions governing the activities of a group or profession are called

Ethics

The primary purpose of tactical EMS is to

Evacuate the patient to the general EMS system

During the reassessment of a stable patient, you should recheck vital signs every 15 minutes. This is an example of:

Evaluating

You can test the oculomotor, trochlear, and abducens nerves by:

Evaluating the patient's extraocular movements

Epidemiologically speaking, an ongoing process that determines the effectiveness, efficiency, and impact of activities related to public health initiatives is called:

Evaluation

Physiologically, the term respiration refers to the:

Exchange of gases at the alveolar-capillary interface

considered to be highly valid.

Experiments using randomized controlled trials​ are:

In which of the following conditions is insertion of a nasogastric tube contraindicated:

Facial fractures

It is not important for you to know the amount of stress you can take before it becomes a problem.

False

Guidelines developed by the National Highway Traffic Safety Administration (NHTSA) for quality improvement in EMS systems include all of the following, EXCEPT:

Financial auditing

Which of the following is the correct order events after an endotracheal tube has been properly inserted?

Inflate the cuff with 5-10cc of air, listen for breath sounds over the epigastrum, listen for equal breath sounds bilaterally, and secure the tube

When using a colormetric end-tidal CO2 detector, the absence of CO2 in exhaled air after 6 breaths indicates the endotracheal tube has been placed:

In the esophagus

The tip of a curved laryngoscope blade is placed correctly:

In the vallecula

Safety inspection, accident prevention, medical screening, and assisting with minor medical care are primary paramedic duties in

Industrial medicine

Which of the following is NOT a reason that a reassessment is important?

It reassures the patient you are taking good care of him.

Safety inspections, accident prevention programs, and medical screening of employees are some of the responsibilities for paramedics working in:

Industrial medicine

Which of the following choices best explains the difference between an EMT-Intermediate and an EMT-Paramedic?

Scope of practice

Analysis of variance

The acronym ANOVA stands for: A) An over-variable analog B) A novel analysis C) Analysis of variance D) Abstract null overaction

Duplex

The type of communication system that works like a telephone is: A) Ultrasimplex B) Multiplex C) Simplex D) Duplex

commonly used to assess data.

The t​ test, ANOVA, and chi square tests​ are:

Standard deviation

Which of the following is a common measure of dispersion? A) Standard deviation B) Confidence interval C) Inferential statistics D) Ordinal data

Which class of hormones is NOT synthesized and secreted by the adrenal cortex? a. Androgen b. Catecholamines c. Mineralocorticoids d. Glucocorticoids

b. Catecholamines

An increased hydrogen ion concentration in the cerebrospinal fluid results in a(n) ________ respiratory rate. decreased unchanged increased erratic

increased

shock signs/symptoms: septic

infection that enters the bloodstream, progressive symptoms- increased cardiac output, toxins causing vasodilation, may appear to be sick, high fever, hypothermia, flushed or pale skin, breathing difficulty, altered lung sounds, altered mental status

One sign of gonorrhea in males is a(n): scant, clear discharge. white, curdlike discharge. profuse, yellow discharge. external pustule.

profuse, yellow discharge.

Exaggerated lumbar concavity

Lordosis (Swayback)

The hypoxic drive is stimulated by:

Low PaO2

Sideways curvature of spine

Scoliosis

the form of shock most often associated with pulmonary edema

cardiogenic shock

What percent of oxygen is delivered by simple face mask?

40 to 60

What is the MOST commonly aspirated material? A) Vomit B) Blood C) Teeth D) Saliva

A) Vomit

10 Vagus assessment

Assess gag and swallowing; Assess vocal quality

___ is your number one priority.

Personal safety

___ trauma patients have very minor injuries or they are dean

Priority 4

Treatment of poisoning emergencies may gernerally consist of any of the following methods, EXCEPT: a. Hemodialysis b. Alkalizing the urine with sodium bicarbonate c. Administering acid solutions to neutralize alkalis is the stomach d. Emptying the stomach.

c. Administering acid solutions to neutralize alkalis is the stomach

Which of the following is NOT typical associated with Type 1 diabetes mellitus. a. Juvenile onset b. Inadequate insulin release from the beta cells of the pancreatic islets c. Insulin dependence d. Obesity

d. Obesity

The maximum flow rate to be used with a nasal cannula is ___ liters per minute. A) 10 B) 6 C) 8 D) 4

B) 6

What is one of the emerging roles and responsibilities of a paramedic in the twenty-first century?

Health promotion

The ability to remain in control is associated with

Leadership

An elderly man is complaining of shortness of breath. You discover pitting edema that is 0.5 to 1 inch deep. This is ________ pitting edema. +1 +3 +2 +4

+3

At standard speed, a 1 mm box on ECG paper represents ________ seconds. 0.12 0.08 0.04 0.1

0.04

When the ECG paper is traveling at the standard rate of 25 mm/sec, a large box in the horizontal direction equals: 0.08 seconds. 0.24 seconds. 0.12 seconds. 0.20 seconds.

0.20 seconds.

There are three large boxes between R waves on an ECG tracing. What is the heart rate? 50 150 75 100

100

Reassessment should include which of the following? 1. Assess effects of interventions. 2. Check vital signs. 3. Assess skin condition. 4. Do detailed physical exam. 5. Check airway patency. 1, 3, and 4 1, 2, 4, and 5 1, 2, 3, and 5 2, 4, and 5

1, 2, 3, and 5

When you are intubating a stoma site, how far beyond the distal cuff (in cm) should you insert the ET tube?

1-2

Correct sequence of steps in critical decision making?

1. Forming a concept 2. Interpreting the data 3. Applying the principles 4. Evaluating results 5. Reflecting on the incident

As adult patient with a pulse who has been intubated due to respiratory arrest should be ventilated _________ times per minute.

10

You are managing a 29-year-old male patient who was shot in the head. There is a gurgling noise in the airway. The patient is breathing spontaneously at 42 breaths per minute, has a heart rate of 110, and a systolic pressure of 102/82. The correct rate of assisted ventilation for this patient is __________ per minute.

10

There are 15 small boxes between R waves on an ECG tracing. What is the heart rate? 20 150 120 100

100

An attempt at endotracheal intubation must not interrupt ventilations for more than _________ seconds.

30

The portable suction device should generate flow rate of __________ liters per minute when the tube is open.

30

For which of the following patients should you conduct a focused physical exam? 35-year-old female roller skater with an ankle fractured after falling 65-year-old male with an altered level of consciousness 15-year-old female pedestrian who was struck by a motor vehicle traveling at 25 mph 22-year-old male with a gunshot wound to the abdomen

35-year-old female roller skater with an ankle fractured after falling

39) How many ATP are produced from glycolysis? A) 2 B) 3 C) 4 D) 5 Answer: C

4

Which of the following choices is the correct sequence of steps in critical decision making? 1. Interpreting the data 2. Evaluating results 3. Reflecting on the incident 4. Forming a concept 5. Applying the principles 1, 2, 4, 5, 3 4, 3, 5, 2, 1 4, 1, 5, 2, 3 3, 4, 5, 1, 2

4, 1, 5, 2, 3

Which of the following is the correct sequence of cardiac electrical activity? 1. AV node 2. Internodal pathways 3. Bundle of His 4. SA node 5. Purkinje fibers 6. Bundle branches 1, 2, 4, 3, 5, 6 4, 1, 2, 3, 6, 5 4, 2, 1, 3, 6, 5 1, 2, 4, 3, 6, 5

4, 2, 1, 3, 6, 5

What is, in liters per minute, the highest flow rate on a demand valve device?

40

A critical patient's vital signs should be reassessed at least every ________ minutes. 5 15 10 20

5

Select the choice below that best describes the sequence of a joint examination. 1. Palpation 2. Passive range of motion 3. Range of motion against resistance 4. Range of motion against gravity 5. Inspection 2, 3, 4, 5, 1 5, 1, 2, 4, 3 1, 5, 2, 4, 3 5, 2, 1, 3, 4

5, 1, 2, 4, 3

Your patient remains unresponsive after a painful stimulus is applied. The family reports that the patient was found unconscious in bed after complaining of a "terrible headache." Which of the following represents the correct sequence of actions in this situation? 1. Obtain a brief history from the family. 2. Conduct a rapid secondary assessment. 3. Perform a detailed exam. 4. Determine vital signs. 5. Perform a primary assessment. 5, 1, 3, 4 5, 1, 2, 4 5, 2, 3, 1 5, 2, 1, 4

5, 2, 1, 4

In which of the following patients would the use of a CombiTube dual lumen airway be used with caution?

59-year-old male with a history of chronic alcoholism who was found in cardiac arrest at a homeless shelter

The maximum flow rate to be used with a nasal cannula is ___________ liters per minute.

6

Without adequate airway maintenance and ventilation, the patient can succumb to brain injury or death in how many minutes?

6-10

water accounts for what percent of total body weight

60%

Immediate intubation should be considered for trauma patients with Glasgow Coma Scale sore of _______ or lower.

8

What is the normal partial pressure of oxygen in the arterial blood?

80 to 100

Moderate hypoxia is indicated with a pulse oximeter reading of ___________ percent

86-91

What type of confidence interval would be appropriate for a research study that is potentially going to change EMS practice?

95

Your patient is a 24-year-old male with a severe closed head injury. He is unresponsive, and his vital signs are unstable. Which of the following is the appropriate receiving facility for this patient?

A Level I trauma center

Which of the following statements concerning Q waves on the ECG is most accurate? A Q wave is significant if it is 0.04 or more seconds wide. The absence of a Q wave is a significant pathophysiological finding. Q waves are not a normal finding on the ECG. A Q wave is significant only in the presence of chest pain.

A Q wave is significant if it is 0.04 or more seconds wide.

Pilot trial

A ____________is undertaken first so you can find unforeseen obstacles to data gathering during participation in a research study. A) Null hypothesis B) Pilot trial C) Principal investigation D) Data analysis

In which of the following situations is a significant amount of carboxyhemoglobin most likely to be present? A patient with COPD who is short of breath with an SpO2 of 90 percent A patient who inhaled anhydrous ammonia fumes A patient found unresponsive in an apartment in which there is a gas furnace A patient who is being treated with nitrites for cyanide poisoning

A patient found unresponsive in an apartment in which there is a gas furnace

During a respiratory assessment, the absence of breath sounds may indicate:

A pneumothorax

You should be prepared to immediately intubate any patient with respiratory distress who is exhibiting:

A pulse oximetry reading of 90 percent or less

All of the following are signs of inadequate breathing in an infant EXCEPT:

A respiratory rate of 32 breaths per minute Signs: Cyanosis Use of accessory muscles Altered mental status

Hypothesis

A specific question that a research study sets out to answer is called the: A) Hypothesis B) Experiment C) Metaanalysis D) Odds ratio

Trunked

A(n) ________ system uses a computer to route transmissions to the first available frequency. A) Repeater B) Digital C) Encrypted D) Trunked

When you are intubating a stoma site, how far beyond the distal cuff (in cm) should you insert the endotracheal tube? A) 1 to 2 B) 3 to 4 C) 2 to 3 D) 4 to 5

A) 1 to 2

What would be the approximate size ET tube to use for a toddler you intend upon intubating? A) 4 B) 5 C) 3 D) 6

A) 4

Without adequate airway maintenance and ventilation, the patient can succumb to brain injury or death in how many minutes? A) 4 B) 10 C) 6 D) 12

A) 4

When providing rescue breathing to an apneic patient using a pocket face mask, each breath should be delivered with ___ of air. A) 6-8 mL/kg B) 10 mL/kg C) 15 mL/kg D) 2-4 mL/kg

A) 6-8 mL/kg

Moderate hypoxia is indicated with a pulse oximeter reading of ___ percent A) 86 to 90 B) 95 to 100 C) 81 to 85 D) 90 to 94

A) 86 to 90

The automatic transport ventilator is contraindicated for all of the following intubated patients, except A) A 17-year-old gunshot victim B) A 4-year-old near-drowning victim C) A 56-year-old in pulmonary edema D) A 34-year-old with adult respiratory distress syndrome

A) A 17-year-old gunshot victim

All of the following are indications for endotracheal intubation, EXCEPT A) A heart rate over 100 B) Respiratory arrest C) Cardiac arrest D) Airway swelling

A) A heart rate over 100

End-tidal colorimetric capnography measures A) Carbon dioxide in exhaled air B) The partial pressure of carbon dioxide in arterial blood C) The amount of carbon dioxide dissolved in plasma D) The percentage of hemoglobin saturated with carbon dioxide

A) Carbon dioxide in exhaled air

Progressively deeper, faster breathing alternating gradually with shallow, slower breathing is called A) Cheyne-Stokes respirations B) Kussmaul's respirations C) Biot's respirations D) Agonal respirations

A) Cheyne-Stokes respirations

Which of the following devices is known as the dual lumen airway? A) Combitube B) Endotracheal tube C) Laryngeal mask airway D) King LTD airway

A) Combitube

Chemoreceptors could be stimulated when which one of the following chemical imbalances occurs? A) Decreased pH B) Increased bicarbonate C) Decreased renin D) Increased nitrogen

A) Decreased pH

32) T lymphocytes are primarily responsible for: A) Directly attacking antigens B) Producing antibodies C) Neutralizing antigens D) Producing antigens Answer: A

A) Directly attacking antigens

When intubating using a lighted stylet, you see a dim, indistinct light in the throat. You have MOST likely intubated the A) Esophagus B) Left mainstem bronchus C) Trachea D) Right mainstem bronchus

A) Esophagus

The simplest airway management technique in a patient without suspected cervical spine injury is A) Head-tilt/chin-lift maneuver B) Modified jaw-thrust maneuver C) Use of an oropharyngeal airway D) Sellick's maneuver

A) Head-tilt/chin-lift maneuver

All of the following are disadvantages of the LMA-Fastrach, EXCEPT A) It can facilitate blind endotracheal intubation B) Inability to decompress the stomach C) absence of sized for patient less than 30 kg D) Tempermental positioning

A) It can facilitate blind endotracheal intubation

One disadvantage of the oropharyngeal airway is A) It doesn't prevent aspiration B) Air can pass around it C) It can be used as a bite block D) It can be sized to the patient

A) It doesn't prevent aspiration

Advantages of a nasopharyngeal airway include all of the following, EXCEPT A) It isolates the trachea B) It can be suctioned through C) It can be inserted with an intact gag reflex D) It can be placed blindly and safely

A) It isolates the trachea

What does the "L" in LEMONS law stand for? A) Look externally B) Leave alone C) Laryngoscopy D) Loose dentures

A) Look externally

The MOST common indication for a surgical cricothyrotomy is A) Massive facial or neck trauma B) Failed rapid sequence intubation C) Status seizures with trismus D) Foreign body airway obstruction

A) Massive facial or neck trauma

Which of the following is NOT and absolute contraindication to rapid sequence airway (RSA)? A) Only one paramedic on scene B) Blunt or penetrating anterior neck trauma C) Upper airway tumor D) Caustic ingestion

A) Only one paramedic on scene

The patient with COPD may benefit from oxygen delivery through a Venturi mask because A) Oxygen concentration is controlled more carefully B) It delivers intermittent positive pressure to the airway C) It both protects the airway and delivers oxygen D) It delivers the highest oxygen concentration possible

A) Oxygen concentration is controlled more carefully

All of the following are indications for RSL, EXCEPT A) Predicted difficult airway B) Hypoxemia despite maximal therapy C) Combativeness secondary to presumed head trauma D) Impending or actual respiratory failure form any cause

A) Predicted difficult airway

Which of the following patients tends to have a decreased oxygen storage capacity? A) Pregnant female B) Patient in shock C) Children D) Patient in pain

A) Pregnant female

Which of the following mechanisms is responsible for hypoxemia in the patient with a pulmonary embolism? A) Pulmonary shunting B) Pulsus paradoxus C) Lower airway obstruction D) Atelectasis

A) Pulmonary shunting

A drop in blood pressure of greater than 10 torr during inspiration is called A) Pulsus paradoxus B) Pulsus obliterans C) Pulsus tardus D) Pulsus alternans

A) Pulsus paradoxus

A technique for intubation by passing a wire through the cricothyroid membrane cephalad through a catheter is called A) Retrograde B) Reverse C) Recumbent D) Rear tracheal

A) Retrograde

Which of the following devices can be used as a simple mechanical airway adjuct--much like an oropharyngeal airway and also as a blind endotracheal tube introducer in situations where laryngoscopy is difficult? A) SALT B) LMA C) ETC D) NPA

A) SALT

A slow, deep, involuntary inspiration followed by a prolonged expiration that hyperinflates the lungs and re-expands atelectatic alveoli is a A) Sigh B) Hiccough C) Grunt D) Sneeze

A) Sigh

A high-pitched inspiratory noise caused by a partial upper airway obstruction is called A) Stridor B) Dysphonia C) Rhonchi D) Wheezing

A) Stridor

One way to improve airway management and ventilation is to make sure that A) The ear-to-sternal notch axis is aligned B) The BVM is squeezed as fully as possible C) Oxygen is not administered as fully as possible D) Never put a patient in the "ramped" position

A) The ear-to-sternal notch axis is aligned

Insertion of an endotracheal tube too far is likely to result in ventilation of A) The right lung only B) The left lung only C) Only the lower lobes of both lungs D) Neither lung

A) The right lung only

When ventilating a patient via a nasal ET, what is one advantage this has over an orally-placed ET? A) The tube cannot be bitten down on B) There is a need for a laryngoscope C) Gag stimulation occurs every time D) Head and neck are extended into position

A) The tube cannot be bitten down on

When correctly placed, the tip of a straight laryngoscope blade should be A) Under the epiglottis B) At the uvula C) At the soft palate D) In the vallecula

A) Under the epiglottis

The loss of muscle tone results in: rigidity. paresthesia. spasticity. flaccidity.

flaccidity.

Which of the following statements regarding needle cricothryotomy is FALSE? A) Upon removal of the needle, the secured catheter allows for direct tracheal suctioning B) High pressure during jet ventilation may cause a pneumothorax C) Perforation of the esophagus may occur if the needle is advanced too far D) It is the least invasive of the surgical procedures and can be initiated quickly

A) Upon removal of the needle, the secured catheter allows for direct tracheal suctioning

The CORRECT sequence for intubating an apneic patient is A) Ventilation, positioning, visualization, tube placement, and verification of tracheal insertion B) Visualization, tube placement, positioning, verification of tracheal insertion, and ventilation C) Tube placement, visualization, positioning, verification of tracheal insertion, and ventilation D) Verification of tracheal insertion, tube placement, visualization, positioning, and ventilation

A) Ventilation, positioning, visualization, tube placement, and verification of tracheal insertion

The amount of gas moved in and out of the respiratory tract in one minute is termed A) Minute volume B) Alveolar volume C) Tidal volume D) Residual volume

A) minute volume

A portable suction device should generate a flow rate of ___ liters per minute when the tube is open. A) 20 B) 30 C) 40 D) 50

B) 30

Which of the following is an example of professional development?

All of the above

____ shows promise in improving outcomes among severely injured crash patients.

AACN

Which of the following may complicate airway management procedures in the pediatric patient?

All of the above are possible complications

Without formal transfer of care to the receiving hospital, paramedics could be charged with:

Abandonment

Which of the following best describes the paramedic's professional obligation?

Acceptance and adherence to a code of professional ethics and etiquette

Which document published in 1966 outlined the deficiencies in prehospital emergency care?

Accidental Death and Disability: The Neglected Disease of Modern Society

Which of the following is the publication that first focused attention on the problem of motor vehicle deaths?

Accidental Death and Disability: The Neglected Disease of Modern Society

Bandwidth

According to a recent study by the National Public Safety Telecommunications​ Council, what is the first challenge that must be addressed before many new EMS communication technologies can be successfully​ introduced? A. Industry acceptance B. Bandwidth C. Funding D. Ambulance alternator capacity

Sudden cardiac arrest and trauma

According to the National EMS Research​ Agenda, which diseases most drive EMS system​ design?

A system ensuring that education programs for paramedics and other EMS personnel levels need minimum guidelines for faculty, facilities, equipment, medical oversight, clinical affiliation and financial stability is

Accreditation

The phases of stress response are

Alarm, resistance, exhaustion

a prospective study

All of the EMS providers at your agency have undergone training in order to participate in a research study. The study will begin on a specific date and end when a certain number of qualifying patients are treated by your department. This would​ be:

In which of the following conditions should you suspect pulmonary shunting?

All of the above

The aspiration of vomitus into the lungs may result in:

All of the above

Which of the following are likely subjects of standing orders or protocols?

All of the above

Which of the following factors is related to an increased likelihood of airway burns?

All of the above

Which of the following increases the risk of foreign body airway obstruction?

All of the above

Wire-based speech-to-monitor transcription

All of the following are other technology applications with broadband implications that the national EMS communications initiatives have suggested, EXCEPT: A) Patient multi-vital-signs monitoring B) Infrared crowd disease detection C) Wire-based speech-to-monitor transcription D) Creation of ad hoc multicomponent patient databases

What control group were the researchers studying?

All of the following are questions to ask when reviewing a study, EXCEPT: A) Was the study type appropriate? B) Was the research peer-reviewed? C) Were the results reported properly? D) What control group were the researchers studying?

Which of the following is NOT an example of the paramedic's responsibility for community involvement?

Allowing citizens to vote on EMS protocols

A physician at the scene of an emergency who is not affiliated with EMS or the EMS service that has been dispatched is

An intervener physician

Direct access to medical consultation is a feature of:

An on-line medical direction

The acronym ANOVA stands for:

Analysis of variance

Critical thinking is a thought process used to:

Analyze and evaluate

With your field diagnosis in mind, you develop a treatment plan for your patient. This part of the critical decision-making process is called:

Applying principles

Which of the following is the most important determinant of ventilatory rate? Arterial PO2 Venous PCO2 Arterial PCO2 Venous PO2

Arterial PCO2

Which of the following serves as an important visual landmark when performing endotracheal intubation under direct intubation?

Arytenoid cartilage

5 Trigeminal (Chewing;Facial Sensation) assessment

Ask patient to hold open mouth while you try to close it and to move the jaw laterally against your hand

7 Facial (Facial Expression) assessment

Ask patient to smile, raise eyebrows, and keep eyes and lips closed while you try to open them

A patient is complaining of abdominal pain, nausea, and vomiting. Which of the following will provide you with the most immediately useful information? Asking about any unusual taste in the mouth Finding out about any unusual stress in the patient's life Checking for discoloration of the tongue Asking about the presence of coffee-grounds emesis

Asking about the presence of coffee-grounds emesis

Which of the following will provide the LEAST information about a patient complaining of abdominal pain? Assessing reaction of the pupils to light Performing a tilt test Checking for unusual breath odors Looking for Cullen's sign

Assessing reaction of the pupils to light

Represented with each contraction of the heart

Atrial Pulse

The National EMS Education Instructional Guidelines are divided into three domains of learning. The affective domain refers to which of the following?

Attitudes, values, and emotions

Which of the following is included in an examination of the cardiovascular system?

Auscultate for carotid bruits

Which of the following is included in an examination of the cardiovascular system? Palpate both carotid arteries simultaneously. Palpate to check for Murphy's sign. Auscultate breath sounds. Auscultate for carotid bruits.

Auscultate for carotid bruits.

The future enhancement of EMS is strongly dependent on:

Availability of quality research

Which of the following is the correct order of events after an endotracheal tube has been properly inserted? A) Inflate the cuff with 5 to 10 cc of air, secure the tube, listen for equal breath sounds bilaterally, and listen for breath sounds over the epigastrium B) Inflate the cuff with 5 to 10 CC of air, auscultate the epigastrium and then the lungs, and secure the tube C) Secure the tube, listen for breath sounds over the epigastrium, listen for equal breath sounds bilaterally, and inflate the cuff with 5 to 10 cc of air D) Inflate the cuff with 5 to 10 cc of air, listen for breath sounds over the epigastrium, listen for equal breath sounds bilaterally, and secure the tube

B) Inflate the cuff with 5 to 10 cc of air, auscultate the epigastrium and then the lungs, and secure the tube

The hypoxic drive is stimulated by A) High PaCO2 B) Low PaO2 C) High PaO2 D) Low PaCO2

B) Low PaO2

Which of the following manual airway maneuvers should be used when you are caring for a patient with a suspected cervical spine injury? A) Sellick's B) Modified jaw-thrust C) Jaw/tongue lift D) Head-tilt/chin-lift

B) Modified jaw-thrust

In which of the following conditions is insertion of a nasogastric tube contraindicated? A) Patient having a gag reflex B) Patient with facial fractures C) Patient awake and alert D) All of the above

B) Patient with facial fractures

The major drawback of mouth-to-mouth ventilations is A) An oxygen level in the rescuer's exhaled air of only 10 to 12 percent B) Potential exposure of the rescuer to body fluids C) Inability to achieve adequate tidal volumes for ventilation D) A high level of carbon dioxide in the rescuer's exhaled air

B) Potential exposure of the rescuer to body fluids

A fine, bubbling sound heard on inspiration and associated with fluid in the alveoli and terminal bronchioles is called A) Bronchovesicular sounds B) Rales (crackles) C) Rhonchi D) Pleural friction

B) Rales (crackles)

You are managing a 29-year-old male patient who was shot in the head. There is a gurgling noise in the airway. The patient is breathing spontaneously at 42 breaths per minute and has a heart rate of 110 and a systolic and diastolic pressure of 102/82. What airway intervention should be performed FIRST on this patient? A) Head-tilt/chin-lift maneuver B) Suctioning the airway C) Insertion of an oropharyngeal airway D) Endotracheal intubation

B) Suctioning the airway

Which of the following statements about manual airway maneuvers is TRUE? A) They require specialized equipment B) They are often neglected by EMTs and paramedics C) They are contraindicated in trauma patients D) They are difficult to learn

B) They are often neglected by EMTs and paramedics

Which of the following is NOT a structure of the upper airway? A) Larynx B) Trachea C) Hypopharynx D) Nasopharynx

B) Trachea

The chemical mediators that are released when mast cells degranulate primarily cause: A) Enhanced cardiac contractility B) Vasodilation C) Depressed pumping action of the heart D) Vasoconstriction

B) Vasodilation

When a portion of the lung is unavailable for gas exchange, yet pulmonary circulation continues in that area of the lung, a condition known as ___ results. A) Pulsus paradoxus B) Ventilation-perfusion mismatch C) Atelectasis D) Eupnea

B) Ventilation-perfusion mismatch

You are managing a 29-year-old male patient who was shot in the head. There is a gurgling noise in the airway. The patient is breathing spontaneously at 42 breaths per minute and has a heart rate of 110 and a systolic and diastolic pressure of 102/82. The correct rate of assisted ventilation for this patient is ___ breaths per minute. A) 6 B) 12 C) 14 D) 18

B)12

What percentage of oxygen is delivered by a simple face mask? A) 60 to 80 B) 95 to 100 C) 40 to 60 D) 80 to 90

C) 40 to 60

What is the normal partial pressure of oxygen in the arterial blood? A) 35 to 45 B) 50 to 75 C) 80 to 100 D) 100 to 150

C) 80 to 100

Saves time and money

Because only the necessary resources are sent on each call, priority dispatching has which of the following benefits? A) Minimizes responder responsibilities B) Saves time and money C) Reduces patient anxiety D) Keeps dispatchers alert

An attempt at endotracheal intubation must not interrupt ventilations for more than ___ seconds. A) 10 B) 15 C) 20 D) 30

C) 20

Pressure in the circulatory system during systole and diastole

Blood Pressure

A 60-year-old man presents with altered mental status. There is no evidence of trauma. Which of the following is most likely to give you specific information about the underlying cause of the patient's condition? Pulse oximetry End-tidal carbon dioxide monitoring Blood glucose determination A Babinski reflex

Blood glucose determination

Which of the following is a traumatic mechanism by which a myocardial injury can be produced? Blunt chest trauma Rapid deceleration Penetrating chest trauma Traumatic brain injury

Blunt chest trauma

A whooshing sound over the carotid artery

Bruits

When coping mechanisms no longer buffer job stressors is

Burnout

Which of the following is a hypertonic solution? A) 0.9 percent sodium chloride B) 0.45 percent sodium chloride C) 1.8 percent sodium chloride D) 0.2 percent sodium chloride Answer: C

C) 1.8 percent sodium chloride

When performing tracheobronchial suctioning, what should be the maximum amount of suction time? A) 15 seconds B) 20 seconds C) 10 seconds D) 5 seconds

C) 10 seconds

Of the normal tidal volume for the average 70 kg adult, what amount of air (in mL) is NOT available for gas exchange? A) 50 B) 100 C) 150 D) 250

C) 150

The tip of the endotracheal tube for the pediatric patient should be inserted no more than ___ cm below the vocal cords. A) 1 to 2 B) 3 to 4 C) 2 to 3 D) 4 to 5

C) 2 to 3

In which of the following patients would the use of a Combitube dual lumen airway be used with caution? A) A 28-year-old male who is in cardiac arrest after his kayak capsized, trapping him under water for several minutes B) A 40-year-old female who has taken an overdose of tricyclic antidepressants and is unresponsive to all stimuli C) A 59-year-old male with a history of chronic alcoholism who was found in cardiac arrest at a homeless shelter D) A 16-year-old female who is unresponsive following a motor vehicle collision in which she was ejected, sustaining traumatic brain injury

C) A 59-year-old male with a history of chronic alcoholism who was found in cardiac arrest at a homeless shelter

The movement of oxygen from the alveoli to the blood in the pulmonary capillaries depends on A) Facilitated transport by way of the hemoglobin molecule B) Active transport of oxygen from an area of lower concentration to an area of higher concentration C) Diffusion of oxygen from an area of higher concentration to an area of lower concentration D) Osmosis of the H2O molecule across the respiratory membrane, where oxygen dissociates from hydrogen

C) Diffusion of oxygen from an area of higher concentration to an area of lower concentration

Which of the following best suggests an esophageal intubation? A) Color change from purple to yellow with a colorimetric ETCO2 detector B) Free return of air when aspirating with an esphageal detector device C) Gurgling sounds over the epigastrium with each ventilation delivered D) Maintaining a pulse oximetry reading of 80 to 85 percent

C) Gurgling sounds over the epigastrium with each ventilation delivered

42) A patient is given antibody therapy in the hospital to prevent infection after exposure to a communicable disease. This is an example of ________ immunity. A) Passive natural B) Active natural C) Passive acquired D) Active acquired Answer: C

C) Passive acquired

The reading obtained by the use of a pulse oximeter reflects the A) Amount of saturated hemoglobin per deciliter of blood B) Amount of oxygen dissolved in the blood C) Ratio of unsaturated hemoglobin to saturated hemoglobin D) Partial pressure of oxygen in capillary blood

C) Ratio of unsaturated hemoglobin to saturated hemoglobin

What is a visualized with a Grade 3 Mallampati score? A) Entire tonsil B) Upper half of the tonsil fossa C) Soft and hard palate D) Only the hard palate

C) Soft and hard palate

When you are suctioning an adult patient, the onset of bradycardia is most likely due to A) Hypercarbia B) Hypoxia C) Stimulation of the vagus nerve D) Increased intracranial pressure

C) Stimulation of the vagus nerve

The preferred depolarizing neuromuscular agent for rapid sequence intubation is A) Pancuronium B) Midazolam C) Succinylcholine D) Vecuronium

C) Succinylcholine

44) Which of the following mechanisms buffers the acidity of the blood the quickest? A) Increased hydrogen ion elimination in the urine B) Protein buffering system C) The carbonic acid-bicarbonate buffer system D) Increased respiratory rate Answer: C

C) The carbonic acid-bicarbonate buffer system

While assessing a 23-year-old male with abdominal pain, you note discoloration around the umbilicus. This is known as:

Cullen's sign

Which of the following certification levels is currently NOT recognized by the National EMS Scope of Practice model?

Critical Care Paramedic

The process of transferring an emergency call to the nearest 911 center is

Call Routing

The paramedic's role has expanded to include ________ because of the need to move ill/injured patients from one health care facility to another for specialized care.

Critical care transport

Your patient is a three-year-old who presents with a fever. His parents state that he has had diarrhea and vomiting for two days. The patient has not eaten in 24 hours. To best evaluate the patient's peripheral perfusion status, you should evaluate:

Capillary refill

End-tidal colormetric capnography measures:

Carbon dioxide in exhaled air

Amount of blood heart ejects in one minute

Cardiac Output (SV x HR)

In which of the following situations would you expect end tidal carbon dioxide levels to be very low, despite patent airway?

Cardiac arrest

Which of the following is affected by a properly working pacemaker? Cardiac rhythm Stroke volume Automaticity Ejection fraction

Cardiac rhythm

Close attention to detail during patient care reflects which of the following professional attributes?

Careful delivery of service

To assess an infant's airway, you should:

Carefully extend the head and neck.

The bifurcation f the trachea is called the:

Carina

Your patient is complaining of numbness and hand pain that wakes him. Of the following, which is most likely? Bursitis Plantar fasciitis Chondromalacia Carpal tunnel syndrome

Carpal tunnel syndrome

Regional radio base stations

Cellular telephone systems use ________ to transmit communications. A) Shortwave technology B) Underground cables C) Satellite technology D) Regional radio base stations

In most states which of the following grants EMS personnel recognition that they have met certain qualifications?

Certification

The process by which an agency or association grants recognition to an individual who has met its qualification is

Certification

3 Oculomotor (Eyelid elevation) assessment

Check PEARL

Progressively deeper, faster breathing alternating gradually with shallow, slower breathing is called:

Cheyne-Stokes respirations

During the cardiac cycle, the S2 heart sound indicates the ____ of the _____ valves.

Closing, aortic and pulmonic

Which of the following devices is known as a dual lumen airway?

CombiTube

Which of the following best describes the practice of evidence-based medicine?

Combining clinical expertise with the best available clinical evidence

The process of exchanging information from one individual to another is

Communication

________ are very important in interpreting the value of the research results.

Confidence intervals

What is a group of subjects who do not have manipulation of the independent variable called?

Control group

The step following prearrival instructions in an EMS response is:

Coordinate call and report incident

Areas of a neurological exam include all of the following EXCEPT:

Cranium Areas: Mental status and speech Reflexes Cranial nerves

Palpate carotid pulse at level of

Cricoid Cartilage

Applying posteriorly directed pressure on the cricoid cartilage to facilitate endotracheal intubation is also referred to as:

Cricoid pressure

Which of the following certification levels is currently NOT recognized by the National EMS Scope of Practice Model?

Critical Care Paramedic

While assessing a 23-year-old man with abdominal pain, you note discoloration around the umbilicus. This is known as: borborygmi. ascites. Cullen's sign. Kernig's sign.

Cullen's sign.

Discoloration over the umbilicus, known as _____, is a(n) _____ indicator of intraabdominal bleeding,

Cullen's, late

Discoloration over the umbilicus, known as ________ sign, is a(n) ________ indicator of intraabdominal bleeding. Cullen's; early Cullen's; late Grey Turner's; late Grey Turner's; early

Cullen's; late

Discoloration around the umbilical area is known as

Cullens Sign

What is, in liters per minute, the highest flow rate on a demand valve device? A) 30 B) 20 C) 15 D) 40

D) 40

30) Which of the following best describes an antibody? A) A toxin released when cells die B) A cell that engulfs and destroys invading pathogens C) A substance secreted by apocrine glands D) A substance produced by B lymphocytes that binds with an antigen Answer: D

D) A substance produced by B lymphocytes that binds with an antigen

Shallow, slow, or infrequent breathing, indicating severe ICP buildup and brain anoxia, is A) Kussmaul breathing B) Grunting C) Biot's respirations D) Agonal breathing

D) Agonal breathing

In which of the following conditions should you suspect pulmonary shunting? A) A foreign body in the right mainstem bronchus B) Pneumonia C) Pulmonary embolism D) All of the above

D) All of the above

The aspiration of vomitus into the lungs may result in A) Tissue damage B) Pulmonary edema C) Pneumonia D) All of the above

D) All of the above

Which of the following increases the risk of foreign body airway obstruction? A) Age B) Alcohol consumption C) Dentures D) All of the above

D) All of the above

Which of the following is a disadvantage of pulse oximetry? A) Tissue hypoxia may be present even with a normal SaO2 reading B) Pulse oximetry cannot differentiate between hemoglobin bound to oxygen and hemoglobin bound to carbon monoxide C) Pulse oximetry gives no information about the amount of carbon dioxide in the blood D) All of the above are disadvantages

D) All of the above are disadvantages

Which of the following may complicate airway management procedures in the pediatric patient? A) The trachea narrows at the cricoid cartilage B) The tongue occupies a greater proportion of space C) The epiglottis is larger and floppier than in an adult D) All of the above are possible complications

D) All of the above are possible complications

In which of the following situations would you expect the end tidal carbon dioxide levels to be very low, despite a patent airway? A) Room air PPV B) Hyperthermia C) Hypoventilation D) Cardiac arrest

D) Cardiac arrest

The bifurcation of the trachea is called the A) Glottic opening B) Hilum C) Cricoid cartilage D) Carina

D) Carina

Which of the following is an advantage of the two-person method of bag-valve-mask ventilation? A) Reduces the risk of gastric distension B) Can provide tidal volumes of 1500 to 1700 mL C) Delivers a higher concentration of oxygen D) Easier to maintain a seal between the patient's face and the mask

D) Easier to maintain a seal between the patient's face and the mask

A(n) ___ may be used to facilitate nasotracheal intubation. A) Uncuffed endotracheal tube B) Malleable stylette C) Lighted stylette D) Endotrol tube

D) Endotrol tube

At the end of exhalation, the intrathoracic pressure is A) Significantly less than atmospheric pressure B) Slightly less than atmospheric pressure C) Greater than atmospheric pressure D) Equal to atmospheric pressure

D) Equal to atmospheric pressure

Physiologically, the term respiration refers to the A) Exchange of gases at the alveolar-capillary interface B) Mechanical process that moves air into and out of the lungs C) Exchange of all gases, nutrients, and wastes at the cellular level D) Exchange of gases at the cellular level

D) Exchange of gases at the cellular level

In an adult, the narrowest part of the airway is the A) Trachea B) Cricoid cartilage C) Hypopharynx D) Glottis

D) Glottis

The tip of a curved laryngoscope blade is placed correctly A) At the junction of the hard and soft palates B) At the glottic opening C) Under the epiglottis D) In the vallecula

D) In the vallecula

Which of the following about the use of a dual lumen airway device is TRUE? A) It is not recommended for trauma patients B) It may be used for adult and pediatric patients C) It may be inserted in patients with a gag reflex D) Insertion requires no direct visualization

D) Insertion requires no direct visualization

As the thoracic cavity begins to expand, the intrathoracic pressure A) Does not change B) Is greater than atmospheric pressure C) Is the same as atmospheric pressure D) Is less than atmospheric pressure

D) Is less than atmospheric pressure

The respiratory rate may be greater than normal because of A) Alcoholic beverage ingestion B) The use of medications such as diazepam or morphine C) Being asleep or nearly asleep D) Metabolic acidosis

D) Metabolic acidosis

45) Your patient is presenting with rapid onset hypotension, tachycardia, and absent radial pulses. Which type of shock is LEAST likely to be the cause? A) Septic B) Cardiogenic C) Hypovolemic D) Neurogenic Answer: D

D) Neurogenic

Which of the following basic airway adjuncts prevents the tongue from falling back to occlude the airway? A) Yankauer catheter B) Laryngeal mask airway C) Nasal cannula D) Oropharyngeal airway

D) Oropharyngeal airway

Which of the following serves as an important visual landmark when performing endotracheal intubation under direct intubation? A) Cricothyroid membrane B) Cricoid cartilage C) Thyroid cartilage D) Posterior cartilages

D) Posterior cartilages

A peak flow meter measures A) Inspiratory reserve volume B) Tidal volume C) The force of expiratory volume D) Respiratory efficacy during forced expiration

D) Respiratory efficacy during forced expiration

One of the primary functions of cortisol as it is released during the stress response is to: A) Inhibit lipolysis B) Promote the release of beta-endorphins C) Potentiate serotonin D) Stimulate gluconeogenesis Answer: D

D) Stimulate gluconeogenesis

Which of the following interferes with ventilation in the presence of a flail segment? A) The ability to generate positive intrathoracic pressure is impaired B) Intrathoracic pressure increases on the affected side C) Intrathoracic pressure decreases on the affected side D) The ability to generate negative intrathoracic pressure is impaired

D) The ability to generate negative intrathoracic pressure is impaired

Which of the following is the most common cause of airway obstruction? A) Foreign bodies B) Food C) Upper airway edema D) The tongue

D) The tongue

A musical, squeaking, or whistling sound that may be heard on inspiration or expiration while auscultating lung fields is known as A) Crackles B) Stridor C) Rhonchi D) Wheezing

D) Wheezing

The first use of a prehospital system of triage and transport took place:

During the Napoleonic Wars

Which of the following findings is NOT a sign of peritoneal irritation?

DeLorenzo's sign

Which of the following findings is NOT a sign of peritoneal irritation? Rebound tenderness DeLorenzo's sign Abdominal pain upon light palpation Abdominal pain when the patient coughs

DeLorenzo's sign

Without proper terminology and verbal communications skills, the receiver will be unable to ________ the message.

Decode

During the return to service, one of the paramedic's chief responsibilities is:

Decontaminating the ambulance

Normal pediatric respiratory and heart rates ______ as children grow older.

Decrease

Cyanosis is caused by increased

Deoxyhemoglobin

The ___ was established following the terrorist attacts of 9/11.

Department of Homeland Security

General radio procedures include all of the following except

Describe in detail your needs and the situation

The first part of the EMS response to an incident is:

Detection and citizen access

What is the first sequence of Communications in EMS response?

Detection and citizen access

The movement of oxygen from the alveoli to the blood in the pulmonary capillaries depends on:

Diffusion of oxygen from as area of high concentration to an area of lower concentration

________ communications are condensed and require a decoder to translate.

Digital

Components of a paramedic's education include all of the following, EXCEPT:

Disciplinary action

The ________ section of a research paper is where the authors interpret their findings and describe their significance.

Discussion

During a musculoskeletal exam, you find redness of the skin over a nontraumatized joint. Which of the following is LEAST likely to be the cause? Gout Rheumatic fever Arthritis Dislocation

Dislocation

Response includes all of the following except

Disposition and transfer

Which of the following may indicate either cardiac tamponade or tension pneumothorax? Hypertension Distended neck veins Subcutaneous emphysema Cushing's reflex

Distended neck veins

A 45-year-old man is alert and oriented, complaining of chest pain. He describes a two-day history of worsening pain, described as a burning sensation below his left breast that radiates across his chest. He states that the pain seems to get better right after eating, but then worsens again. Medical history includes myocardial infarction, hypertension, and type 2 diabetes. His medications include Inderal, Vasotech, Glucotrol, and Lopid. Physical examination reveals cool, dry skin; clear lungs; and no JVD or peripheral edema. HR = 102, BP = 132/86, RR = 12, SaO2 = 98%. The patient is on oxygen by nasal cannula at 4 liters per minute. Which of the following should be done next? Do a 12-lead ECG. Start precautionary IV of normal saline and transport. Give 0.4 mg nitroglycerin sublingually. Give 81 mg aspirin by mouth.

Do a 12-lead ECG.

This type of study does not allow the subjects or the experimenters to know who is in the control group and who is not.

Double-blind

___ uses a different frequency to transmit and receive.

Duplex

The loss of muscle tone results in:

Flaccidity

Pleural Effusion

Fluid in pleural space

Which of the following guidelines should be observed when examining a one- to three-year-old child?

Focus on the vital areas as indicated by the chief complaint.

Which of the following guidelines should be observed when examining a one- to three-year-old child? Restrain the child if necessary. Perform a comprehensive head-to-toe exam on all children in this age group. Do not distract the toddler with toys. Focus on the vital areas as indicated by the chief complaint.

Focus on the vital areas as indicated by the chief complaint.

Which of the following guidelines should be observed when examining a one-to-three-year-old child?

Focus on the vital areas as indicated by the chief complaint.

The emergency medical dispatcher has just finished interrogating a caller. The next step would be for the dispatcher to:

Follow established guidelines to determine the appropriate level of response

The first step in critical thinking it to:

Form a concept

After considering the information you have collected, you determine the most likely cause of your patient's condition. This step in the critical decision-making process is called:

Forming a concept

The number of times per minute a radio wave oscillates is its:

Frequency

Which of the following statements about a paramedic's skill competency is true?

Frequent review of infrequently used skills is critical to ensure competency.

One area of EMS that will rely especially heavily on research is:

Funding

In an adult, the narrowest part of the airway is:

Glottis

1 Olfactory (Smell) assessment

Have patient identify familiar smell with eyes closed

4 Trochlear (EOM-eyes turning downward and laterally) assessment

Have patient looking down and in

6 Abducens (EOM) assessment

Have patient move eyes side to side

2 Optic (Vision) assessment

Have patient read from card one eye at a time

11 Spinal Accessory (Shoulder and Head movement) Assessment

Have patient shrug shoulders and move head side to side

Romberg Test

Have patient standing with eyes closed and feet together for 20-30 seconds Positive-Lose Balance Negative-Not Lose Balance

12 Hypoglosseal (Tongue Movement) Assessment

Have patient stick out tongue and move it internally

With regard to medical authorization, which of the following best describes the role of a paramedic in providing prehospital care?

He relies on the delegated practice of a licensed physician medical director.

The simplest airway management technique in patient without suspected cervical spine injury is the:

Head-tilt/chin lift maneuver

Roles of the paramedic include

Health care

Which one of the following roles does EMS have in public health when providing health care screenings and vaccinations, providing health education, and targeting high-risk populations in an effort to ensure that they are receiving needed medical care?

Health promotion

Which branch of EMS responds to both scene calls and interfaculty transfers?

Helicopter EMS

Shining a light onto the iris from the lateral side could cause a shadow on the medial side if the patient is suffering from:

Hemianopsia

During spontaneous breathing, which of the following reflexes prevents overinflation of the lungs?

Hering-Breuer

What procedure can you perform to determine additional signs of carpal tunnel syndrome? Hold the patient's wrist in acute flexion for 60 seconds. Ask the patient to gently wave his wrists for 60 seconds. Ask the patient to extend his fingers forward for 60 seconds. Hold the patient's wrist in acute extension for 60 seconds.

Hold the patient's wrist in acute flexion for 60 seconds.

Assisting with field triage destination and transportation decisions

How can AACN improve outcomes with seriously injured patients? A) Assisting with field triage destination and transportation decisions B) Increasing response times by prehospital care providers C) Predicting the unlikelihood of serious injury among vehicle occupants D) Increasing the time it takes for patients to receive definitive trauma care

Researchers may use certain data elements to assess patient outcomes.

How can your patient care reports be used for research​ purposes? A. You can use the PCR for teaching purposes. B. Researchers may use certain data elements to assess patient outcomes. C. All information from PCRs are fed into a national database. D. The​ patient's primary care physician will receive a copy.

Medical reports emphasize history; trauma reports emphasize injuries and exam findings.

How do reports to the base physician differ for trauma and medical patients? A) It is less important to include ETA for medical patients. B) Only trauma patients require a description of the scene. C) Medical reports describe a chief complaint while trauma reports do not. D) Medical reports emphasize history; trauma reports emphasize injuries and exam findings.

When you percuss a patients chest and hear a loud, booming, low pitched sound, its known as:

Hyper inflated lung field

During a neurologic exam you ask your patient to stick out his tongue. It deviates to the right. This may indicate a lesion to the patient's ____ nerve.

Hypoglossal

During a neurologic exam you ask your patient to stick out his tongue. It deviates to the right. This may indicate a lesion to the patient's ________ nerve.

Hypoglossal

A specific question that a research study sets out to answer is called the:

Hypothesis

ETCO2 is recorded during phase ________ of the capnogram. I IV II III

III

To inspect and examine the anus, a paramedic should place the patient

left or right lateral recumbent

The mean is especially useful when the data are what statisticians call "normally distributed." This means:

If you graphed the data, it would form a shape similar to a bell curve

When a patient is misinterpreting what is happening

Illusion

Echo procedure

Immediate repetition of medical orders received during radio communications is known as the: A) Verification protocol B) Echo procedure C) Standard format D) Response algorithm

In 1980 the revision "KKK-A-1822A" involved:

Improving ambulance electrical systems by designing a low-amp lighting system to replace antiquated light bars and beacons

Which of the following can be determined by lead II cardiac monitoring? Axis deviation Chamber enlargement Impulse conduction time Presence of a myocardial infarction, but not its location

Impulse conduction time

Where are the results of a scientific study published, regardless of the outcome of the study?

In a peer review journal

1, 2, and 3

In addition to giving emergency instruction to the caller, a dispatcher can also use prearrival instructions to: 1. Comfort a distressed caller 2. Elicit additional information 3. Provide emotional support 4. Coordinate responders A) 1 and 2 only B) 1, 3, and 4 C) 1, 2, and 3 D) All of the above

Observational

In many situations it is unethical to withhold treatment from a group simply for the purposes of experimentation. As a result a(n) _________ study must be done. A) Observational B) Quasiexperimental C) Retrospective D) Prospective

PSAP, EMD

In most systems 911 callers are first routed to the ________, which then sends the call to ________. A) EMS-C, PSAP B) PSAP, base station C) EMD, PAAP D) PSAP, EMD

patient outcome information.

In order to conduct useful EMS​ research, the researchers must have access​ to:

the application of scientific research.

In paramedic​ practice, many traditional EMS treatments have been abandoned or refined as a result​ of:

Professionalism

In terms of written communication, a neat and complete PCR is a good indicator of which of the following paramedic traits? A) Professionalism B) Compassion C) Promotion suitability D) General intelligence level

All of the above

Information on the prehospital care reports may be used by which of the following in certain circumstances? A) Hospital staff B) Insurance claims departments C) Attorneys D) All of the above

The two types of education in EMS include

Initial, Continuing

Common strategies that include child safety seat classes, bicycle safety training, drunk driving education programs, smoking prevention, and swimming pool safety programs fall under which public health role of EMS?

Injury prevention

A way for paramedics to determine appropriate injury prevention programs for their community is by using:

Injury risk surveys

A(n) ___________ is the ongoing systematic collection, analysis, and interpretation of injury data essential to the planning, implementation, and evaluation of public health practice.

Injury surveillance program

Sequence for examining chest

Inspect Palpate Percuss Auscultate

As you survey the scene, you are gathering information about your patient and the scene. This step in the critical decision-making process is called:

Interpreting the data

A physician not affiliated with the responding EMS system who takes part in patient care with system paramedics at an emergency scene is called a(n):

Intervening physician

Public Safety and Homeland Security Bureau.

In​ 2008, the FCC established a new office to handle public safety​ issues, known as​ the:

In ____ muscles are worked through their range of motion.

Isotonic exercise

Which of the following best describes why a focused history is important in responsive medical patients? It allows treatments to be initiated immediately. It supports diagnostic impression. It helps to formulate a field diagnosis. It identifies signs of medical complications.

It helps to formulate a field diagnosis.

Which of the following best characterizes the professional development of a paramedic?

It is a career-long pursuit.

Which of the following statements is FALSE regarding the S3 heart sound? It is commonly heard in children. It can develop with ventricular failure or ventricular volume overload. It is a high-pitched sound heard at the end of diastole. It is the "dee" part of the "lub-dub-dee" cadence.

It is a high-pitched sound heard at the end of diastole.

Which of the following statements is FALSE regarding the S3 heart sound?

It is a high-pitched sound heard at the end of diastole. TRUE: It is commonly heard in children It is the "Dee" part of the "lub-dub-dee" cadence It can develop with ventricular failure or ventricular volume overload

Which of the following best describes the initial education program of the paramedic?

It is the base and beginning of one's professional education, not the end.

Which of the following is NOT a reason that a reassessment is important? It allows you to assess the effects of interventions. It allows you to detect trends in the patient's status. It determines changes in the patient's conditions. It reassures the patient you are taking good care of him.

It reassures the patient you are taking good care of him.

Which of the following about the use of a dual lumen airway device is true?

It requires no neck hyperextension

Why is a detailed physical exam rarely performed on critical trauma patients in the prehospital setting? It is used only on medical patients. It usually yields little immediately useful information. The rapid trauma assessment performs the same function. It produces too much patient anxiety.

It usually yields little immediately useful information.

Using knowledge and experience to diagnose patients and plan their treatment is called clinical:

Judgment

Sounds auscultated when assessing the blood pressure

Korotkoff sounds

A condition that makes posterior chest and lung examination difficult is thoracic:

Kyphoscoliosis

Forward rounding of back

Kyphosis (Hunchback)

During abdominal exam, painful or tender areas should be assessed when?

Last

During the abdominal exam, areas that are known to be painful or tender should be examined:

Last

After the primary assessment of an unresponsive medical patient with stable vital signs, which of the following is the position in which the patient should be placed? Lateral recumbent Supine Prone Semi-Fowler's

Lateral recumbent

Which of the following is NOT a category within public health laws?

Laws that deal with the incidence and prevalence of disease in large populations

Which of the following professional characteristics is displayed in a paramedic's self-confidence, inner strength, ability to communicate, and willingness to make a decision?

Leadership

Which of the following arteries supplies blood primarily to the left ventricle and posterior wall? Posterior descending Marginal Right coronary Left coronary

Left coronary

Hospitals that provide definitive trauma care for all kinds of trauma are

Level I only

Hospital that serves communities that can provide prompt assessment, resuscitation, emergencies operations, but does not have definitive trauma care is a

Level III

Although termed certification by many states, the governmental agency's permission to engage in a profession actually constitutes:

Licensure

The process by which a governmental agency grants permission to engage in a given occupation to an applicant who has attained the degree of competency required to ensure the public's protection is called:

Licensure

The process by which a governmental agency grants permission to engage in a given trade of profession is

Licensure

Maintaining a complete and accurate written PCR is essential to all of the following except

Maintain HIPA

When evaluating any patient's genitalia, a paramedic must always:

Maintain privacy

Which of the following is a normal finding when examining a patient's neck? Visible lymph nodes Subcutaneous emphysema Movement of the thyroid gland during swallowing Tracheal tugging

Movement of the thyroid gland during swallowing

The MOST common indication for a surgical cricothyrotomy is:

Massive facial or neck trauma

The difference between mean and median is which of the following?

Mean is calculated by adding values, then dividing the sum by the number of values involved; median is calculated by putting the values into numerical order and finding the middle value.

the mean value

Measures of central tendency​ include: A. the mean value. B. the range between highest and lowest values. C. the difference between the highest and lowest values. D. the sum of all values collected.

Prehospital care reports

Medical audits, research, and policy changes can improve patient care through information collected from: A) Prehospital care reports B) Crew activity logs C) Vehicle maintenance logs D) Disciplinary action records

Paramedics may function only under the direction and license of the EMS system's

Medical director

How do reports to the base physician differ for trauma and medical patients?

Medical reports emphasize history; trauma reports emphasize injuries and exam findings.

The standard of cleanliness in the ambulance is

Medically clean

The respiratory rate may be greater than normal because of:

Metabolic acidosis

This section of a research paper describes exactly how the authors conducted the study, including what population they wished to study, how subjects were selected (and excluded), and what intervention was performed.

Methods

Which of the following manual airway maneuvers should be sued when you are caring for a patient with a suspected cervical spine injury?

Modified jaw-thrust

Voice and data simultaneously

Multiplex systems transmit: A) Voice simultaneously in two ways B) On the same frequency as they receive C) Using digital encryption technology D) Voice and data simultaneously

Which of the following body systems would you gain the least information about during an abdominal exam?

Musculoskeletal

Which of the following body systems would you gain the least information about during an abdominal exam? Cardiovascular Musculoskeletal Reproductive Digestive

Musculoskeletal

Your patient is a 62-year-old woman with a history of COPD. She is slightly more short of breath than usual today. She is alert and speaks three to four words between breaths. Her SpO2 is 93 percent on room air. Which of the following devices should be used for the administration of oxygen to this patient? BVM Nasal cannula CPAP Nonrebreather mask

Nasal cannula

The EMT Code of Ethics was established by which of the following groups?

National Association of EMTs (NAEMT)

This type of study has less validity than an RCT, but it has utility in some circumstances. The problem in this study design is that there is an increased chance that the study groups will be different.

Nonrandomized controlled trials

A 45-year-old patient with abdominal pain in the right lower quadrant. Upon auscultation of bowel sounds, you hear high-pitched gurgles and clicks lasting around 10 seconds. This patient may have:

Normal sounds

Which of the following best suggests an esophageal intubation?

Not seeing the tip of the endotracheal tube pass through the vocal cords

In many situations it is unethical to withhold treatment from a group simply for the purposes of experimentation. As a result a(n) ________ study must be done.

Observational

What is the first step of the scientific method?

Observe and ask questions

What is one way to read the patient?

Observe the level of consciousness

What is one way to read the patient? Observe the level of consciousness. Address life threats. Observe the mechanism of injury. Obtain the history from family members.

Observe the level of consciousness.

You arrive at a location and begin to read the scene by:

Observing the immediate surroundings

multiple localized control centers

Of the following​ list, which concept does not apply to the communications component of an EMS​ system? A. medical communications B. communications software C. multiple localized control centers D. citizen access

Medical policies, procedures, and practices that a system medical director has established in advance of a call are called:

Off-line medical oversight

According to US Dept. of AG dietary guidelines, you should make ___ of your plate fruits and vegetables

One half

Which of the following best describes the technique for palpating the abdomen for large or deep masses?

One hand over another, pushing down slowly

Which of the following best describes the technique for palpating the abdomen for large or deep masses? One hand pushing deeply in a quick, smooth motion One finger pressing firmly against the abdominal wall One hand over another, pushing down slowly Both hands side by side over the midline of the abdomen

One hand over another, pushing down slowly

Unit and provider information

One of the first bits of information to be transmitted to the base physician is the: A) Estimated time of arrival to the hospital B) Request for specific orders C) Unit and provider information D) Treatments already rendered

Which of the following basic airway adjuncts prevents the tongue from falling back to occlude the airway?

Oropharyngeal airway

This type of research can help determine whether a procedure, drug, treatment or similar strategy actually improves patient outcomes.

Outcomes-based

The patient with COPD may benefit from oxygen delivery through a Venturi mask because:

Oxygen concentration is controlled more carefully

Your patient is a 44-year-old woman, alert and oriented, in moderate distress and complaining of difficulty breathing. She gives a one-week history of fever and malaise, with shortness of breath developing three days ago. She also has left-sided chest pain with deep inspiration and a "phlegmy" cough. Physical examination reveals hot, pale, dry skin, and rhonchi and crackles throughout the left lung. The right lung sounds are clear. HR = 134, BP = 88/64, RR = 24, SaO2 = 92%. She has a history of two previous myocardial infarctions and takes nitroglycerin as needed. Which of the following is the best course of prehospital management? Oxygen via nonrebreather mask and IV of NS with fluid challenge Albuterol via nebulizer with 100 percent oxygen and IV of NS KVO Oxygen by nonrebreather mask; IV of NS KVO; and furosemide, 40 mg IV Endotracheal intubation, ventilation with supplemental oxygen, IV of NS KVO, nebulized albuterol and Atrovent, and corticosteroids IV

Oxygen via nonrebreather mask and IV of NS with fluid challenge

This particular statistic reflects only the odds of seeing the results of a particular piece of research if the study hypothesis is true.

P value

In an infant the heart rate is best detected by either palpating the brachial artery or

Palpating the femoral pulse

Physical Exam Techniques

Palpation Auscultation Inspection

Community paramedicine

Paramedics and other EMTs becoming affordable in rural communities and also helping to fill gaps in primary health care services is a concept of: A) Community paramedicine B) Creation of ad hoc databases C) Medical quality video D) Public safety answering points

For safety at a roadway incident, it is not appropriate to do the following

Park on opposite side of the street from the crash

Which of the following is NOT part of the commitment that an EMS provider needs to have to prevent illness and injury?

Parking the unit in the safest place to load the patient, then directing traffic

The emerging roles and responsibilities of the paramedic do not include

Participation in injury and illness prevention

Defending patients, protecting them, and, in general, acting in their best interests are all examples of:

Patient advocacy

Which of the following is one of the paramedic's primary responsibilities?

Patient disposition

Which of the following is NOT one of the paramedic's primary responsibilities?

Patient follow-up

Paramedics can improve their knowledge and skills through an evaluation by others of equal rank and skills, known as:

Peer review

An example of a quality improvement activity in EMS is:

Peer review of patient care

Paramedics treat patients with the same techniques as other clinicians, except that they:

Perform these procedures in uncontrollable and unpredictable environments.

Which of the following is LEAST important to the paramedic when evaluating a patient's skin in an emergency situation? Temperature Pigmentation Moisture Color

Pigmentation

A ________ is undertaken first so you can find unforeseen obstacles to data gathering during participation in a research study.

Pilot trial

Percussion

Place hand on a body part, then sharply taps a distal knuckle with the tip of another finger

Babinski Reflex

Plantar Flexion

Squeaking or grating sounds of chest lining rubbing together

Pleural Rub

Increased fremitus over part of the patient's chest wall may indicate

Pneumonia

Which of the following is not an important principle of lifting?

Position load far from your body

While you are assessing for a Babinski response, the patient's big toe dorsiflexes and the other toes fan out. This indicates a _____ response which is ______.

Positive, abnormal

The major drawback of mouth-to-mouth ventilation is:

Potential exposure of the rescuer to communicable disease

The dispatcher's directions to the caller for appropriate emergency measures are known as:

Prearrival instructions

Amount of blood returned to the heart from the body

Preload

The paramedic is responsible for determining the mechanism of injury or nature of the illness when carrying out which of the primary responsibilities?

Scene size-up

The person who is overseeing a study and is familiar with requirements for informed consent is the:

Principal investigator

An occupation in which the practitioners have a competence in a specialized body of knowledge or skills that has been recognized by some organization or agency is called a:

Profession

In terms of written communication, a neat and complete PCR is a good indicator of which of the following paramedic traits?

Professionalism

Which of the following is NOT part of a paramedic's administrative duties?

Promoting community health

When comparing and contrasting between a retrospective and prospective study, which of the following is a TRUE statement?

Prospective studies have greater validity than retrospective studies.

A droop in the eyelids is known as

Ptosis

Which of the following mechanisms is responsible for hypoxemia in the patient with a pulmonary embolism?

Pulmonary shunting

Which of the following is LEAST urgent when evaluating a patient suspected of suffering an acute myocardial infarction? Pulse oximetry ECG Blood pressure Breath sounds

Pulse oximetry

Which of the following statements about pulse oximetry is FALSE? It may be difficult to obtain a reading in patients experiencing peripheral vasoconstriction. Pulse oximetry should be used on all patients with respiratory complaints. Pulse oximetry values can be expected to decrease within seconds in cases of developing hypoxia. Oxygen saturation is the percentage of hemoglobin that is bound with some molecular structure.

Pulse oximetry values can be expected to decrease within seconds in cases of developing hypoxia.

A drop in blood pressure of greater than 10 torr during inspiration is called:

Pulsus paradoxus

The total duration of ventricular depolarization is represented by the ________ on the ECG. PQ interval R-R interval QRS duration QT interval

QRS duration

___ research describes phenomena in numbers

Quantitative

________ describes phenomena in numbers.

Quantitative research

What is the difference between qualitative and quantitative research?

Quantitative research is designed to determine the relationship between one thing and another and describe it with numbers; qualitative research primarily relies on collection of nonnumeric data.

Inspiratory nonmusical popping sound heard during inspiration

Rales (Crackles)

A fine, bubbling, sound heard on inspiration and associated with fluid in the alveoli and terminal bronchioles is called:

Rales (crackles)

A peak flow meter measures:

Rate of airflow during forced expiration

The reading obtained by the use of a pulse oximeter reflects the:

Ratio of unsaturated hemoglobin to saturated hemoglobin

The process by which an agency in one state grants automatic certification or licensing to a paramedic who is certified or licensed by an agency in another state is called:

Reciprocity

The act of receiviang a comparable certification of licensure from another state or agency is known as

Reciprosity

Petechiae (Skin Lesions)

Reddish-purple spots less than 0.5cm in diameter

After a call you discuss the patient with the ED physician, comparing diagnoses and care plans. This is an example of:

Reflecting

Types of disposition include all of the following except

Refusal of care

Entering a person's name and relevant information in a particular record maintained by an organization is best described as:

Registration

Establishing terminology

Regulation of communications at the federal level includes all of the following, EXCEPT: A) Licensing technical personnel B) Establishing technical standards C) Establishing terminology D) Allocating radio frequencies

Type of murmur is produced when blood is pushed back through a valve, most commonly the mitral valve

Regurgitation

Problem is identified, observations and experiments used to construct and test the hypothesis thought to solve it

Scientific method

This process is one by which scientists endeavor to construct an accurate representation of the world that is reliable, consistent, and nonarbitrary.

Scientific method

Your patient is a 16-year-old male who attempted suicide. He is unconscious and apneic, lying supine on a garage floor. The family states that they found the patient unconscious in the front seat of a car that was running in an enclosed garage. HR = 70, BP = 100/60, RR = 0. In addition to an IV of normal saline, which of the following is the most appropriate action? Remove the patient from the garage, initiate BVM ventilations with 100 percent oxygen, intubate, and transport to the nearest facility. Remove the patient from the garage, intubate, and transport to the nearest hospital. Intubate, remove the patient from the garage, and transport to a hospital with a hyperbaric chamber. Remove the patient from the garage, initiate BVM ventilations with 100 percent oxygen, intubate, and transport to a hospital with a hyperbaric chamber.

Remove the patient from the garage, initiate BVM ventilations with 100 percent oxygen, intubate, and transport to a hospital with a hyperbaric chamber.

You are en route to a hospital with a patient who appears stable following a motor vehicle collision. To detect changes in the patient's condition, you should do which of the following during transport? Neurological assessment Detailed physical examination Rapid trauma assessment Repeat the primary assessment

Repeat the primary assessment

Which of the following situations is NOT typically addressed by EMS system protocols?

Requirements for continuing education

Many traditional EMS treatments and practices have been abandoned or refined because of:

Research

Which of the following National Highway Traffic Safety Administration (NHTSA) elements of EMS systems deals with issues of equal access to acceptable emergency care for all patients?

Resource management

Your three-year-old patient opens her eyes and responds when you speak to her. Her mental status is best described as:

Responsive to verbal stimuli

Changing or stopping interventions that are NOT working is an example of:

Revising the management plan

Continuous sounds with lower pitch and snoring quality

Rhonchi

Jugular venous pressure approximates the patient's ____ pressure.

Right atrial

A patient who loses his balance while standing with his eyes closed and feet together for 20 to 30 seconds is exhibiting a positive:

Romberg test

A patient who loses his balance while standing with his eyes closed and feet together for 20 to 30 seconds is exhibiting a positive: Romberg test. Babinski's reflex. Kehr's sign. tilt test.

Romberg test.

8 Vestibularcochlear (Hearing;Equilibrium) assessment

Rub your fingers near each ear

Guidelines that must be followed for permitting a new medication, process, or procedure to be used in EMS

Rules of evidence

Heart sound is caused by the mitral and tricuspid valve closing

S1

Which of the following sounds is produced when the tricuspid and mitral valves close during systole?

S1

Which of the following sounds is produced when the tricuspid and mitral valves close during systole? S4 S1 S2 S3

S1

Heart sound is caused by the semi-lunar valves closing

S2

Heart sound is representative of heart failure or volume overload

S3

Heart tone known as an atrial gallop

S4

Difference between the values obtained and those that actually exist in the total population

Sampling error

Heart sound S2 may be best heard at the end of systole by auscultating over the ______ intercostal space.

Second

Heart sound S2 may be heard at the end of systole by auscultating over the ________ intercostal space.

Second

The paramedic who routinely fails to complete duties and paperwork in a timely manner is most likely lacking which of the following professional attributes?

Self-motivation

Specialized terms for radio use enhance communications by:

Shortening airtime and transmitting thoughts quickly

Candidiasis

Shows lower abdominal and genital pain; white, curdle vaginal discharge43

Systems that transmit and receive on the same frequency are ________ systems.

Simplex

___ transmissions use the same frequencies to transmit and receive

Simplex

For you to assess for jugular vein distention, your patient should be ____ at a _____-degree.

Sitting, 45

Which of the following is an abnormal finding of the pupils? Constriction when focusing on a near object Less than 2 mm difference in size with normal reaction to light Both pupils constricting when a penlight is shone in one eye Slow but equal reaction to light

Slow but equal reaction to light

Which of the following is an abnormal finding of the pupils?

Slow but equal reaction to light.

Shortening airtime and transmitting thoughts quickly

Specialized terms for radio use enhance communications by: A) Lengthening airtime and transmitting thoughts quickly B) Shortening airtime and transmitting thoughts quickly C) Rationing airtime and transmitting thoughts reasonably accurately D) Shortening airtime and transmitting thoughts reasonably accurately

Injury prevention, definitive treatment of minor injuries, and lifesaving treatments of major injuries are primary paramedic duties in

Sports medicine

In order to link its EMS and public health systems, a community must have which of the following in place?

Stakeholders having a role in the planning process

Which of the following is a common measure of dispersion?

Standard deviation

Your patient is a 73-year-old man who is sitting in a recliner, cyanotic, pulseless, and apneic. The patient's skin is cool and dry. The patient's wife last saw him an hour and a half ago. Which of the following should you do first? Attach the monitor/defibrillator. Inform the patient's wife that he is dead and nothing can be done for him. Start CPR. Check for rigor mortis.

Start CPR.

Type of murmur is produced by narrowing of a valve, most commonly the aortic valve

Stenosis

Which of the following would result in an increased respiratory rate? Stimulation of baroreceptors by an increase of PCO2 A decrease of cerebrospinal fluid PO2 An increase of cerebrospinal fluid pH Stimulation of chemoreceptors by an increase of PCO2

Stimulation of chemoreceptors by an increase of PCO2

When you are suctioning an adult patient, the onset of bradycardia is most likely due to:

Stimulation of the vagus

Amount of blood heart ejects in one beat

Stoke volume

The ability to critically evaluate a patient's condition and formulate a treatment plan rests on all of the following EXCEPT:

Strictly following protocols TRUE: An excellent working knowledge of anatomy and physiology Identifying and dealing with medical ambiguity The ability to focus on large amounts of data

A high-pitched inspiratory noise caused by a partial upper airway obstruction is called:

Stridor

Which of the following best describes the technique of evaluating plantar reflexes? Strike the triceps along the posterior aspect of the distal humerus. Have the patient dorsiflex the foot, then strike the Achilles tendon. Tap the patellar tendon just below the edge of the patella. Stroke the lateral aspect of the sole of the foot from heel to ball, curving medially.

Stroke the lateral aspect of the sole of the foot from heel to ball, curving medially.

A tear in the tracheobronchial tree or a pneumothorax can be characterized by ________ in the neck.

Subcutaneous emphysema

The preferred depolarizing neuromuscular agent for rapid sequence intubation is:

Succinylcholine

What airway intervention should be performed first on this patient?

Suctioning the airway

Which of the following best describes the correct positioning of a patient for an abdominal exam? Lateral recumbent Supine with the head flat and legs extended Supine with the head and knees supported by pillows Semi-Fowler's position

Supine with the head and knees supported by pillows

Which of the following best describes the correct positioning of a patient for an abdominal exam?

Supine with the head flat and legs extended

The main duties of the dispatcher, after sending the responders and providing prearrival instructions, include:

Supporting and coordinating

Simplex

Systems that transmit and receive on the same frequency are ________ systems. A) Multiplex B) Duplex C) Simplex D) Trunked

Ventricular repolarization of the myocardium is represented by the: beginning of the QRS complex to the apex of the T wave. isoelectric line. T wave. ST segment.

T wave.

You are assessing a 34-year-old female who appears to be in hypovolemic shock. You would expect her pulse to be:

Tachycardia and Weak

A time shortly after an emergency or near emergency when the patient and observers remain acutely aware of what has or may have happened and are especially receptive

Teachable moment

To gain informed consent

Telling participants the inclusion and exclusion criteria for enrolling patients in the study, the effect of the study on patient care in general, and the risks and potential benefits to patients in the study helps: A) To gain informed consent B) To get EMS concessions C) To identify the PI D) To determine how to publish

Which of the following pieces of legislation had a significant NEGATIVE impact on EMS funding?

The Consolidated Omnibus Budget Reconciliation Act (COBRA)

protect human subjects in research.

The Institutional Review Board​ (IRB) exists​ to:

Abstract

The ________ is a brief paragraph that summarizes the need for the study, the research methods used, and the results encountered. A) Abstract B) Mode C) Hypothesis D) Parameter

Emergency medical dispatcher

The ________, the public's first contact with the EMS system, plays a crucial role in every EMS response. A) Emergency medical technician B) Emergency medical dispatcher C) Paramedic D) First responder

Discussion

The ____________ section of a research paper is where the authors interpret their findings and describe their significance. A) Summary B) Results C) Methods D) Discussion

Which of the following interferes with ventilation in the presence of a flail segment?

The ability to generate negative intrathoracic pressure is impaired.

Public safety answering point

The acronym PSAP stands for: A) Paramedic system answering point B) Prehospital safety and prevention C) Public safety answering point D) Public safety agency paramedics

Mean is calculated by adding values, then dividing the sum by the number of values involved; median is calculated by putting the values into numerical order and finding the middle value.

The difference between mean and median is which of the following? A) Mean is calculated by adding values, then dividing the sum by the number of values involved; median is calculated by putting the values into numerical order and finding the middle value. B) There is no statistical difference between the mean and the median. C) Median is calculated by adding values, then dividing the sum by the number of values involved; mean is calculated by putting the values into numerical order and finding the middle value. D) Mean is the value of the variance within a study, and median is a representation of the standard deviation.

Which of the following best describes the pulse pressure? The sum of systolic and diastolic blood pressures The difference between systolic and diastolic blood pressures The systolic pressure minus one-third of the diastolic pressure The diastolic pressure plus one-third of systolic pressure

The difference between systolic and diastolic blood pressures

Prearrival instructions

The dispatcher's directions to the caller for appropriate emergency measures are known as: A) Enhanced 911 service B) Priority dispatching C) Prearrival instructions D) Call coordination

Follow established guidelines to determine the appropriate level of response

The emergency medical dispatcher has just finished interrogating a caller. The next step would be for the dispatcher to: A) Follow established guidelines to determine the appropriate level of response B) Call the patient's insurance company for preapproval C) Contact the patient's physician to determine the legitimacy of the call D) Send a first responder engine company and paramedics

FCC

The federal agency that controls and regulates nongovernmental communications is the: A) FCC B) FAC C) FCA D) FAA

Detection and citizen access

The first part of the EMS response to an incident is: A) Discussion with medical direction B) Prearrival instructions C) Detection and citizen access D) Call coordination

the Nuremburg Code.

The first regulation to guide ethical practice in human research was the result​ of:

Abstract

The first thing to appear after the title and names of the authors of a research paper is the: A) Introduction B) Summary C) Abstract D) Results

Helsinki Declaration

The fundamental principles of the​ __________ are: respect for the​ individual, ability of the subject to make an informed decision about participating in the​ research, and assurance by the researcher that the​ patient's safety will be protected.

title and authors.

The information included in a research paper prior to the abstract is​ the:

Few areas of the United States provided adequate prehospital care until:

The late 1960s

Supporting and coordinating

The main duties of the dispatcher, after sending the responders and providing prearrival instructions, include: A) Transferring communications B) Discussing medical direction C) Directing the crew to an appropriate receiving facility D) Supporting and coordinating

the states.

The majority of the financial burden to modernize communications equipment for EMS falls​ on:

If you graphed the data, it would form a shape similar to a bell curve

The mean is especially useful when the data are what statisticians call "normally distributed." This means: A) If you graphed the data, it would form a shape similar to a bell curve B) Values are placed into numerical order, and the middle value is determined C) The data falls within the "fiftieth percentile" D) The distribution of a group is how spread out it is, or how dispersed the data is

What is semantics?

The meaning of words

multiplex technology

The medical director for your agency is mandating simultaneous transmission of your verbal report and telemetry data on all cardiac patients. This will​ require:

Metaanalysis of randomized controlled trials

The most valid of all the specific study types is: A) Cross-sectional study B) Metaanalysis of randomized controlled trials C) Expert opinions, editorials, rational conjecture D) Bench research

Which of the following characteristics is most similar between the prehospital setting and other medical care environments? The variety of uncontrolled environments Unpredictable circumstances The absence of laboratory results to guide treatment The need to establish an airway early in patient care

The need to establish an airway early in patient care

Which of the following characteristics is most similar between the prehospital setting and other medical care environments?

The need to establish an airway early in patient care.

Frequency

The number of times per minute a radio wave oscillates is its: A) Frequency B) Amplitude C) Wavelength D) Bandwidth

Maintaining paramedic certification through seeking and reporting continuing education is the responsibility of:

The paramedic

A 12-lead ECG that reveals slight ST segment elevation; Q waves in leads II, III, and aVF; and ST elevation in V1 and V2 most indicate which of the following? The patient is experiencing ischemia and injury in her left ventricle and septum. The patient is experiencing myocardial injury in her lateral wall. The patient is experiencing myocardial ischemia in her inferior wall that is extending into the septum. The patient has had a myocardial infarction in her inferior wall and is experiencing ischemia extending into the right ventricle.

The patient has had a myocardial infarction in her inferior wall and is experiencing ischemia extending into the right ventricle.

Principal investigator

The person who is overseeing a study and is familiar with requirements for informed consent is the: A) Controller B) Institutional review foreman C) Principal investigator D) Abstract editor

1970s

The practice of notifying a receiving hospital of an ambulance's impending arrival began in the: A) 1960s B) 1970s C) 1980s D) 1990s

Medical

The prehospital care report (PCR) is a written record of events that includes administrative and ________ information. A) Weather B) Medical C) Vehicle maintenance D) Extraneous

Public record

The prehospital care report is all of the following, EXCEPT a(n): A) Legal record of the incident B) Public record C) Indicator of professionalism D) Part of the patient's permanent medical record

Priority dispatching

The process of interrogating callers with medical questions to elicit information and then determine the proper level of response is known as: A) Telephone triage B) Acuity assignment C) Call queueing D) Priority dispatching

Insertion of an endotracheal tube too far is likely to result in ventilation of:

The right lung only

Coordinate call and report incident

The step following prearrival instructions in an EMS response is: A) Coordinate call and report incident B) Discuss medical direction C) Transfer communications D) Take call and effect emergency response

Which of the following is the most common cause of airway obstruction?

The tongue

Giving a formal verbal briefing

The transfer of care to the receiving facility staff should always include: A) Having the physician examine the report B) Giving a formal verbal briefing C) Having the nurse sign the prehospital chart D) Immediately notifying the dispatcher that the unit is back in service

Which branch of EMS responds to both scene calls and interfaculty transfers?

The transport of critically ill or injured patients is referred to as

is likely to be more beneficial in rural areas.

The use of video streaming in EMS to transmit patient images to medical​ direction: A. is already being used in more than 30 states. B. is being challenged on the grounds that it violates HIPAA. C. is being fully supported by the national EMS community. D. is likely to be more beneficial in rural areas.

Treatments rendered

The verbal report to the receiving hospital should always include vital information, chief complaint, and: A) Personal physician's information B) Treatments rendered C) Next-of-kin name and telephone number D) EMS unit status

Which of the following statements about manual airway maneuvers is true?

They are often neglected by EMTs and paramedics

all of the options are correct.

The​ FCC's primary functions​ include: A. spot-checking base stations and dispatch centers for appropriate licenses and records. B. licensing and regulating the technical personnel who repair and operate radio equipment. C. monitoring frequencies to ensure appropriate usage. D. all of the options are correct.

main findings of the study.

The​ summary, or​ conclusion, is a very brief recap of​ the:

You have applied a CO-oximeter to your patient, and it is displaying an SpCO of 15 percent. Which of the following is the most appropriate interpretation of this finding? This is consistent with mild carbon monoxide poisoning. This is a normal reading for a smoker and nothing to worry about. This is a normal reading for a nonsmoker and nothing to worry about. This is consistent with a fatal level of carbon monoxide poisoning.

This is consistent with mild carbon monoxide poisoning.

You are working in the ED, caring for a 55-year-old female patient with a long history of COPD. She is more short of breath today than usual, and states that she has an increased cough. She has a tympanic temperature of 99.8°F. You have drawn arterial blood gases with the patient on room air; when the report comes back, it shows that the patient has a PaO2 of 92 mmHg. Which of the following is most likely? The patient is critically hypoxic and requires assisted ventilation. The lab performed the test incorrectly. This is the typical value for this patient. You have inadvertently drawn a venous sample.

This is the typical value for this patient.

P value

This particular statistic reflects only the odds of seeing the results of a particular piece of research if the study hypothesis is true. A) Nominal data B) P value C) Chi square test D) T test

Scientific method

This process is one by which scientists endeavor to construct an accurate representation of the world that is reliable, consistent, and nonarbitrary. A) Case series B) Random sampling C) Iterative process D) Scientific method

Methods

This section of a research paper describes exactly how the authors conducted the study, including what population they wished to study, how subjects were selected (and excluded), and what intervention was performed: A) Introduction B) Discussion C) Methods D) Results

Outcomes-based

This type of research can help determine whether a procedure, drug, treatment or similar strategy actually improves patient outcomes. A) Bench B) Outcomes-based C) Mixed D) Qualitative

Double-blind

This type of study does not allow the subjects or the experimenters to know who is in the control group and who is not. A) Quasiexperimental B) Cross-sectional C) Double-blind D) Single-blind

Nonrandomized controlled trials

This type of study has less validity than an RCT, but it has utility in some circumstances. The problem in this study design is that there is an increased chance that the study groups will be different. A) Cohort study B) Nonrandomized controlled trials C) Cross-sectional study D) Case series

How has public health improved both the quality of life and the life span of humankind?

Through research Through epidemiology Through surveillance All of the above

Sending multiple levels of emergency care personnel to the same incident is called a:

Tiered response

Which of the following is a disadvantage of pulse oximetry?

Tissue hypoxia may be present even with a normal SaO2 reading

Telling participants the inclusion and exclusion criteria for enrolling patients in the study, the effect of the study on patient care in general, and the risks and potential benefits to patients in the study helps:

To gain informed consent

9 Glossopharyngeal (Gagging and Swallowing) assessment

Touch back of throat with sterile tongue depressor or have patient swallow

Which of the following is NOT a structure of the upper airway?

Trachea

What is the last step in the sequence of communications during an EMS response?

Transferring communications

As a rule, which of the following services should be present in trauma systems within an EMS system?

Trauma center

The verbal report to the receiving hospital should always include vital information, chief complaint, and:

Treatments rendered

During ventricular systole the ventricles contract while the:

Tricuspid and mitral valves close

Defensive strategies while responding to stress are helpful short term.

True

In EMS, an intervention must be affordable and teachable.

True

Morbidity is the rate or incidence of a disease.

True

___ frequencies are typically used for their ability to transmit through concrete and steel.

UHF

When correctly placed, the tip of a straight laryngoscope blade should be:

Under the epiglottis

they penetrate concrete and steel well and are less susceptible to interference.

Units operating in a large city may transmit on ultrahigh frequency​ (UHF) radio frequencies​ because:

Which of the following patient conditions requires immediate transport? Unresponsiveness to painful stimuli after ALS interventions Narcotic overdose Neck pain following a motor vehicle collision Wrist fracture with diminished distal sensation

Unresponsiveness to painful stimuli after ALS interventions

To test an extremity for pain sensation, you should:

Use a sharp object, and have the patient tell you if he feels a sharp or dull sensation

The receiver must be able to decode the message

Using terminology widely accepted by the emergency services and medical community when giving a report to emergency department personnel is important because: A) Semantics enhance effective communication B) The receiver must be able to decode the message C) Verbal communication is the preferred medium D) "10-codes" are meaningless to physicians

The anterior surface of the heart is best viewed by ECG leads: aVR, aVL, aVF. V1-V4. II, III, aVF. I, aVL.

V1-V4.

The lead to the left of the sternum at the fourth intercostal space is: V2. V4. V1. V3.

V2

____ frequencies are used due to their ability to transmit over various terrain and longer distances.

VHF

When you ask the patient to puff out her cheeks, show her upper and lower teeth, and raise her eyebrows, which cranial nerve are you evaluating?

VII

When you ask the patient to puff out her cheeks, show her upper and lower teeth, and raise her eyebrows, which cranial nerve are you evaluating? VI VII V IX

VII

Whether or how well a study supports the conclusions is referred to as:

Validity

When a portion of the lung is unavailable for gas exchange, yet pulmonary circulation continues in that area of the lung, a condition known as _____________ results.

Ventilation-perfusion mismatch

Tactile Fremitus

Vibrations through the chest wall as patient speaks

Which of the following findings is abnormal in a cardiovascular assessment?

Vibrations when palpating the carotid artery

Which of the following findings is abnormal in a cardiovascular assessment? Splitting of the S2 heart sound in children Visible pulsation of the PMI at the fifth intercostal space, medial to the left midclavicular line Jugular venous distension in the supine position Vibrations when palpating the carotid artery

Vibrations when palpating the carotid artery

Multiplex systems transmit:

Voice and data simultaneously

All of the following are questions to ask when reviewing a study, EXCEPT:

What control group were the researchers studying?

face to face

What form of communication is used most often by EMS​ providers?

Control group

What is a group of subjects who do not have manipulation of the independent variable called? A) Subject group B) Experimental group C) Treatment group D) Control group

The meaning of words

What is semantics? A) A standard format for delivering information B) The meaning of words C) Effective communications D) Process of exchanging information between individuals

Quantitative research is designed to determine the relationship between one thing and another and describe it with numbers; qualitative research primarily relies on collection of nonnumeric data.

What is the difference between qualitative and quantitative research? A) Quantitative research uses a control group; qualitative research does not use a control group. B) Quantitative research is designed to determine the relationship between one thing and another and describe it with numbers; qualitative research primarily relies on collection of nonnumeric data. C) Quantitative research does not use a control group; qualitative research does use a control group. D) Qualitative research is designed to determine the relationship between one thing and another and describe it with numbers; quantitative research primarily relies on collection of nonnumeric data.

Observe and ask questions

What is the first step of the scientific method? A) Observe and ask questions B) Construct a hypothesis C) Conduct research and collect data D) Test the hypothesis

research

What is the key to maintaining a focus on improving community health in a​ cost-conscious environment?

Transferring communications

What is the last step in the sequence of communications during an EMS response? A) Calling 911 B) Transferring communications C) Emergency medical dispatch D) Citizen detection and access

95

What type of confidence interval would be appropriate for a research study that is potentially going to change EMS practice? A) 80 B) 85 C) 90 D) 95

Continuous high pitched musical sounds similar to a whistle

Wheeze

A musical, squeaking, or whistling sound that may be heard on inspiration or expiration while auscultating lung fields is known as:

Wheezing

Trunking

When a radio transmission comes in, a computer routes it to the first available frequency. This is known as: A) Trunking B) A computer C) A TTY D) A facsimile machine

"affirmative" and​ "negative."

When communicating via​ radio, you should​ use: A. "yes" and​ "no." B. "sure" and​ "ok." C. "please" and​ "thank you." D. ​"affirmative" and​ "negative."

Prospective studies have greater validity than retrospective studies.

When comparing and contrasting between a retrospective and prospective study, which of the following is a TRUE statement? A) Prospective studies are generally less expensive than retrospective methodologies. B) There is more chance for the introduction of bias in the data gathering for prospective studies. C) Prospective studies have greater validity than retrospective studies. D) Retrospective studies use a research form or instrument specifically designed for the study

All of the above

When preparing a written report, you must remain aware that it is a(n) ________ and a part of your patient's permanent medical record. A) Representation of the care you provided B) Legal record C) Document used for continuous quality improvement D) All of the above

Knowing ________ is NOT part of the paramedic's responsibility to be prepared prior to an EMS call.

When to call for additional help

In a peer review journal

Where are the results of a scientific study published, regardless of the outcome of the study? A) In a medical director paper B) In an EMS trade magazine C) In a university-based magazine D) In a peer review journal

Validity

Whether or how well a study supports the conclusions is referred to as: A) Variance B) Validity C) Bias D) Ordinal data

Duplex transmission

Which of the following allows simultaneous​ two-way communication? A.Simplex transmission B. Duplex transmission C. Uniplex frequencies D. 800 MHz frequencies

To coordinate and implement the treatment plan

Which of the following best explains the reason for the need to communicate effectively with other responders? A) To coordinate and implement the treatment plan B) To summon EMS through a PSAP C) To make sure you can recover your equipment afterward D) To exchange career information

the abstract section

Which of the following briefly summarizes a research​ paper? A. The limitations section B. The methods section C. The references section D. The abstract section

Use codes that are part of your neighboring EMS system

Which of the following is NOT a guideline for effective radio use? A) Press the transmit button for one second before speaking. B) Use codes that are part of your neighboring EMS system. C) Speak in a normal pitch, keeping your voice free of emotion. D) Do not waste airtime with unnecessary information.

IRB

Which of the following is a committee that​ approves, monitors, and reviews all research involving human test​ subjects? A. OHRP B. IRB C. HHS D. NIH

Hypoparathyroidism leads to decreased: a. Calcium levels b. Metabolic rate c. Thyroxine (T4) levels d. Growth hormone levels.

a. Calcium levels

A standard set of medically approved questions

Which of the following is most effective in helping the dispatcher get information from distressed callers? A) Having supervisors get on the line with the callers B) A standard set of medically approved questions C) Long pauses to allow the callers to collect their thoughts D) Loud and authoritative language

Communication

Which of the following is the key link in the chain that results in the best possible patient outcome? A) Conceptualization B) Communication C) Coordination D) Confrontation

Automatic crash notification

Which of the following is the newest technology related to EMS communications and​ response? A. Enhanced 911 B. Automatic crash notification C. Public safety answering points D. Global positioning systems

A sloppy PCR implies sloppy care

Which of the following is true about a sloppy patient care​ report? A. A PCR only needs to be legible to the author. B. A sloppy PCR indicates it was completed in a hurry and is acceptable only if the patient was a high priority. C. A sloppy PCR is not a reflection on the​ patient's care. D. A sloppy PCR implies sloppy care

Neither the patient nor doctor knows if the patient receives the drug or a placebo.

Which of the following is true of a double blind study involving a new​ medication? A. Both the doctor and the patient decide together if the new medication should be given. B. Neither the patient nor doctor knows if the patient receives the drug or a placebo. C. The patient chooses to receive either the new medication or the placebo. D. The doctor chooses the new medication or the​ placebo, but the patient is unaware.

They can only transmit or​ receive, but not both simultaneously.

Which of the following is true regarding simplex radio​ communications? A. They should not be used when an incident command structure is in place. B. They are not useful for any EMS communications. C. They can only transmit or​ receive, but not both simultaneously. D. They allow simultaneous​ two-way communications, but not data transmission

they present challenges that the EMS system is still trying to address

Which of the following is true regarding wireless 911​ calls? A. 911 calls from wireless devices are always routed to the nearest 911 center. B. 911 calls from a mobile phone will always provide the incident location automatically. C. The challenges of wireless 911 calls have been resolved. D. They present challenges that the EMS system is still trying to address.

UHF

Which of the following radio waves can penetrate through concrete and steel? A) UHF B) VHF C) ICT D) GIS

Federal Communications Commission

Which of the following regulates all nongovernmental communication in the United​ States? A. National Broadcast Agency B. National Association of Radio Communications C. Federal Communications Commission D. U.S. Department of Transportation

All researchers

Who is responsible for ensuring each study participant provides informed​ consent?

To confirm understanding

Why is it most important to repeat orders from medical​ direction?

All of the following are other technology applications with broadband implications that the national EMS communications initiatives have suggested, EXCEPT:

Wire-based speech-to-monitor transcription

Experimental

Within a(n) ____________study, subjects are randomized into either the treatment group or the control group; this type is considered among the most valid of studies. A) Experimental B) Quasiexperimental C) Observational D) Retrospective

Abandonment

Without formal transfer of care to the receiving hospital, paramedics could be charged with: A) Malpractice B) Abuse C) Abandonment D) Neglect

Decode

Without proper terminology and verbal communications skills, the receiver will be unable to ________ the message. A) Decode B) Resend C) Encode D) Receive

what group you will draw your conclusions from.

You are assisting with an EMS research project and have defined the population. This means you​ know:

Give the patient's chief complaint

You are giving a report to the base physician. After stating the patient's age, sex, and weight, you should next: A) Give the patient's chief complaint B) Give the hospital the ETA C) Relay treatments already rendered D) Request specific orders

Ask a research librarian

You are having trouble finding scholarly articles related to your research topic. Which of the following is the best way to get​ assistance? A. Use an online search engine. B. Ask a research librarian. C. Contact your EMS instructor. D. Ask other healthcare providers.

The study was approved by the IRB.

You have located an article online and are assessing its reliability. Which of the following is a good indication it is a scholarly​ article? A. It is available online. B. It has at least two authors. C. It includes at least three sources. D. The study was approved by the IRB.

median.

You have placed the values of all data in numerical order and identified the middle value. This is known as​ the:

not relay confidential patient information.

Your​ agency's radio frequencies may be monitored by the public and media. You​ should: A. advise listeners to get off the frequency before transmitting patient information. B. not relay confidential patient information. C. use​ 10-codes to communicate essential information. D. identify all patients as high priority for liability reasons.

Digital

________ communications are condensed and require a decoder to translate. A) Cellular B) Analog C) Digital D) Telephone

Quantitative research

_____________ describes phenomena in numbers. A) Prospective research B) Qualitative research C) Mixed research D) Quantitative research

Confidence intervals

______________ are very important in interpreting the value of the research results. A) Measures of central tendency B) Standard deviations C) Qualitative statistics D) Confidence intervals

dysplasia

a change in cell size, shape, or appearance caused by an external stressor

atrophy

a decrease in cell size resulting from a decreased workload

cause of metabolic acidosis

a deficiency of bicarbonate (HCO-3) in the body. usually results from an increase in metabolic acids- primarily through anaerobic metabolism. low oxygen stores.

oncotic force

a form of osmotic pressure exerted by the large protein particles, or colloids, present in blood plasma

organ system

a group of organs that work together (ex: cardiovascular system)

organ

a group of tissues functioning together (ex: heart)

antibody

a substance produced by B lymphocytes in response to the presence of a foreign antigen that will combine with and control or destroy the antigen, thus preventing infection

electrolyte

a substance that, in water, separates into electrically charged particles

Which of the following is an acceptable antidote for a specific poisoning? a. Acetylcysteine (Mucomyst) for acetaminophen overdose b. Sodium bicarbonate for muriatic acid ingestion c. Amyl nitirte, sodium nitrite and sodium thiosulfate for organoposphate exposure d. Naloxone for diazepam overdose

a. Acetylcysteine (Mucomyst) for acetaminophen overdose

In ventricular fibrillation, admiodarone should be: a. Administered at 300 mg dose IVP b. Administered as 150 mg dose IVP over 3 to 4 minutes c. Mixed in a 50 mL bag and infused of 8 to 10 minutes d. Mixed in a 250 mL bag and infused at a rate of 2 to 4 mg/min.

a. Administered at 300 mg dose IVP

The narcotic nalbuphine (Nubain) not only binds to opiate receptor sites to provide analgesia, it prevents other opioids, such as morphine sulfate, from binding to these sites, lessening respiratory depression. These characteristics make the drug a(n): a. Agonist-antagonist b. Competitive antagonist c. Competitive agonist d. Agonist

a. Agonist-antagonist

An important distinction between analgesics and anesthetics is: a. Anesthetics block all sensations b. Anesthetics can only be administered intravenously c. Analgesics produce unconsciousness is high doses d. Analgesics are reversible, whereas anesthetics are not

a. Anesthetics block all sensations

Drugs that affect blood coagulation help to prevent injuries such as myocardial infarction and cerebrovascular accidents. These drugs are classified as: a. Antiplatelets, anticoagulants, and thrombolytics b. Thrombolytics, plasma blockers, and antiplatelets c. Thrombolytics, plasma blockers, and anticoagulants d. Anticoagulants, plasma enhancers, and thrombolytics

a. Antiplatelets, anticoagulants, and thrombolytics

Physostigmine (Antilirium) is a useful antidote for which drug overdoes? a. Atropine b. Dopamine c. Norepinephrine d. Epinephrine

a. Atropine

Which of the following diseases is characterized by hypersecretion of adrenocorticotropic hormone leading to hyperglycemia, obesity, hypertension, and electrolyte imbalances? a. Cushing's disease b. Gigantism c. Hashimoto's disease d. Addison's disease

a. Cushing's disease

Your patient is a 28-year-old female who is complaining of a rash on her abdomen and tightness in her chest. She ate shellfish approximately 2 hours ago. The symptoms first appeared 1 hour ago and have not worsened. Her vital signs are normal and lung sounds are clear with good tidal volume. The patient is without any further complain. Appropriate initial pharmacological treatment for this patient may include: a. Diphenhydramine 25mg IM b. Solu-Medrol 50mg IVP c. Epinephrine 0.3mg 1:10,000 IVP d. Cortizone Topical Cream

a. Diphenhydramine 25mg IM

Benzodiazepines and barbiturates achieve their sedation effects by ________ in the CNS. a. Enhancing the GABA receptors site b. Depolarizing the neural membranes c. Blocking the GABA receptor sites d. Blocking the dopaminiergic receptor sites.

a. Enhancing the GABA receptors site

Which ovarian hormone is used pharmacologically to treat breast cancer and osteoporosis? a. Estrogen b. Progesterone c. Luteinizing hormone (LH) d. Follicle stimulating hormone (FSH)

a. Estrogen

As antiemetics, phenothiazine (Compzaine, Phenergan) have which of the following undesirable side effects? a. Extrapyramidal effects and sedation b. Headache and diarrhea c. Hallucinations d. Eurporia

a. Extrapyramidal effects and sedation

Glucagon will affect blood glucose levels by: a. Inducing glycogenolysis and gluconeogenesis b. Inducing the storage of glucose into the liver and skeletal muscle through gluconeogenesis c. Enhancing the absorption of glucose from the gastrointestinal tract d. Enabling the rapid absorption of glucose into the cells.

a. Inducing glycogenolysis and gluconeogenesis

When treating patients with suspected hypoglycemia, which of the following should be of greatest concern regarding the administration of 50 percent of dextrose in water. a. Infiltration of the IV during the drug administration b. The administration of oxygen before any other therapies c. Ensuring that the blood glucose specimen is is obtained before administering the drug d. Confirming the patient has a history of diabetes.

a. Infiltration of the IV during the drug administration

Which of the following is the primary hormone that regulates glucose metabolism? a. Insulin b. Glycogen c. Thyroxine d. Levothyroxine

a. Insulin

While obtaining a medical history on a patient complaining of general malaise, the patient informs you that she is currently taking a prescription thyroid hormone. Failure to comply with the prescription of this medication may lead to changes in: a. Metabolism b. Cardiac re-polarization c. Cardiac depolarization d. Urination and water retention.

a. Metabolism

Current treatment regimens for peptic ulcer disease may include all of the following, EXCEPT: a. Nonsteroidal antinflammatory drugs (NSAIDs) b. H2 receptor antagonsists c. Proton pump inhibitors d. Antacids

a. Nonsteroidal antinflammatory drugs (NSAIDs)

Multiple transmitters are involved in the vomitting reflex, including all of the following EXCEPT: a. Norepinephrine b. Acetylcholine c. Dopamine d. Serotonin

a. Norepinephrine

Tricyclic antidepressants achieve their therapeutic effect by: a. Prolonging the duration of norepinephrine and serotonin at the receptor sites. b. Enhancing degradation of norepinephrine and serotonins at the synapse. c. Blocking the release of norepinephrine and serotonin at the synapse d. Blocking the release of acetylcholine at the synapse

a. Prolonging the duration of norepinephrine and serotonin at the receptor sites.

Theophylline is one example of the group of repiratory medications known as xanthines. Unlike other repiratory medications, this group of medications: a. Promotes diuresis. d. Relaxes the cardiac muscle. c. Stimulates bronchial smooth muscle. d. Decreases diaphragmatic contractility.

a. Promotes diuresis.

Promethazine (Phenergan) is primarily used in the prehospital setting to: a. Relieve nausea b. induce sedation c. counteract allergies d. neutralize stomach acid.

a. Relieve nausea

A college student, in an effort to improve test scores, takes a drug Intelliboost, which enhances cortical function s by increasing dopamine release in the brain, He also is taking a cold remedy, Sneezeless, which inhibits the chemical breakdown of dopamine at al receptor sites. Which of the following best describes the drub interation? a. Sneezeless potentiates the effects of Intelliboost. b. Sneezeless has a cumulative effect. c. Sneezeless and Intelliboost have a synergistic effect d. Sneezeless and intelliboost have an additive effect.

a. Sneezeless potentiates the effects of Intelliboost.

Methylphenidate (Ritalin), a drug used to treat attention deficit hyperactivity disorder (ADHD), achieves its therapeutic action by _______ the CNS, thereby causing _______ and allowing for greater concentration and focus. a. Stimulating, heightened awareness and arousal b. Depressing, sedation c. Stimulation, paradoxical sedation d. Depressing, relaxation

a. Stimulating, heightened awareness and arousal

Benzodiazepines are safer than barbiturates in seizure patients because: a. The effect of benzodiazepines is limited by the amount of endogenous GABA in the CNS b. Benzodiazepines cannot cause hypotension or respiratory depression c. The therapeutic dose of barbiturates is very close to the lethal dose d. Benzodiazepines cause a counteraction by inducing high levels of dopamine in the CNS

a. The effect of benzodiazepines is limited by the amount of endogenous GABA in the CNS

Your crew is called to the home of a 56-year-old male who is found unconscious in his apartment. In an attempt to gain some insight into the patient's past medical history, you review the prescription bottles found in the bathroom. One medication you find is labeled "Retrovir." What assumption might you make based on this medication? a. The patient may have HIV b. The patient may have tuberculosis c. The patient may have a recent history of coronary bypass surgery d. The patient may have an electrolyte imbalance secondary to hemodialysis

a. The patient may have HIV

One sign of an adverse drug reaction is the formation of raised, itchy bumps. The term given to this finding is: a. Urticarial b. Nephritis c. Capillaritis d. Epidermatitis

a. Urticarial

Stimulation of the histamine (H1) receptors often results in: a. Vasodilation and increased vascular permeability b. Increased gastric acid release c. Inhibition of antibody production d. Bronchodilation

a. Vasodilation and increased vascular permeability

Emergency medications that affect the heart and blood vessels do so via the: a. autonomic nervous system b. somatic nervous system c. involuntary nervous system d. parasympathetic nervous system

a. autonomic nervous system

In general, diuretic medications: a. lower blood pressure by causing the kidneys to remove excess amounts of salt and water from the body. b. increase cardiac preload and stroke volume by increasing the force of myocardial contraction. c. lower blood pressure throught their effects of negative inotropy, negative chronotropy and negative dromotropy. d. cause a mild increase in blood pressure by lowering the quantity of sodium and calcium ions in the body.

a. lower blood pressure by causing the kidneys to remove excess amounts of salt and water from the body.

The first thing to appear after the title and names of the authors of a research paper is the:

abstract

An important aspect of the reassessment is to monitor and evaluate all of the following EXCEPT the: transport priority. accumulation of charges for care. effectiveness of interventions. vital signs.

accumulation of charges for care.

Artifacts such as muscle tremors can make assessment of cardiac rhythm difficult because: an artifact can cause the pacer to fire prematurely. the QRS complexes may be obscured by the pacer spikes. an artifact can obscure the underlying rhythm. the pacemaker may confuse the artifact with an electrical impulse and not fire.

an artifact can obscure the underlying rhythm.

The fourth heart sound, S4, is: an atrial gallop sound. the "dub" of "dee-lub-dub." a high-pitched sound at the base of the heart. a ventricular gallop sound.

an atrial gallop sound.

hypertrophy

an increase in cell size resulting from an increased workload

hyperplasia

an increase in the number of cells resulting from an increased workload

Lung perfusion depends on all of the following EXCEPT: intact pulmonary capillaries. efficient pumping of blood by the heart. adequate blood volume. an intact alveolar membrane.

an intact alveolar membrane.

anion

an ion with a negative charge

cation

an ion with a positive charge

hemoglobin

an iron-based pigment present in red blood cells that binds with oxygen and transports it to the cells

Critical thinking is a thought process used to: analyze and evaluate. establish credibility with an emergency physician. focus on a situation's most important aspect. defend one's actions in a CQI review process.

analyze and evaluate.

With your field diagnosis in mind, you develop a treatment plan for your patient. This part of the critical decision-making process is called: forming a concept. applying principles. interpreting the data. evaluating the results.

applying principles.

normal body pH

approximately 7.4 (7.35-7.45)

The most important factor in determining the respiratory rate is: -arterial pO2. -alveolar pO2. -arterial pCO2. -alveolar pCO2.

arterial pCO2.

Most carbon dioxide from cellular metabolism reaches the alveoli by being transported: as carbonic anhydrase. as bicarbonate ion. dissolved in plasma. bound to hemoglobin.

as bicarbonate ion.

In an infant, the heart rate is best detected by either palpating the brachial artery or: auscultating the carotid pulse. palpating the carotid pulse. palpating the femoral pulse. auscultating the apical pulse.

auscultating the apical pulse.

The initial adult IV dose of adenosine is: a. 6 mg/kg Rapid IVP b. 6 mg Rapid IVP c. 12 mg/kg Slow IVP d. 12 mg IV over 10 minutes.

b. 6 mg Rapid IVP

Diuretics are commonly used with other antihypertensive medications to control blood pressure effectively. Diuretics results in: a. A loss of excess sugar and water from the body via urination b. A loss of excess salt and water in the body by renal excretion c. An increase in salt and decrease in water from the body by renal excretion d. An increase in sugar and a decrease in water form the body by hemodialysis.

b. A loss of excess salt and water in the body by renal excretion

The recommended dose of vasopressin in cardiac arrest due to pulseless VT of VF is: a. 1 mEq/kg IV bolus repeated once in 5 minutes. b. A single IV bolus does of 40 units. c. 40 units IV bolus repeated once in 5 minutes with 1/2 of of the initial dose. d. 2 to 4 mg IV bolus admnistered in 2 mg increments, titrated to desired response.

b. A single IV bolus does of 40 units.

Sympathetic nervous system stimulation that results in vasoconstriction is most likely affecting: a. Beta 2 receptor sites b. Alpha 1 receptor sites c. Beta 1 receptor sites d. Alpha 2 receptor sites

b. Alpha 1 receptor sites

Which classification of drugs affecting the autonomic nervous system would be best suited to treat a patient with spinal shock? a. Alpha antagonist b. Alpha agonist c. Beta antagonist d. Beta agonist

b. Alpha agonist

While norepinephrine will bind to both postganglionic adrenergic receptor sites, it has an affinity for: a. Beta receptors b. Alpha receptors c. Nicotinic receptors d. Muscarinic receptors

b. Alpha receptors

Which of the following is a physiologic effect of tricyclic antidepressants? a. Selectively block serotonin from being reabsorbed in the brain. b. Blocks norepinephrine and serotonin from being reabsorbed in the brain c. Blocks epinephrine from reaching its receptors, resulting in bradycardia d. Blocks alpha-2 receptors, resulting in decreased secretion of norepinephrine

b. Blocks norepinephrine and serotonin from being reabsorbed in the brain

Symptoms of hyperthyroidism include all of the following, EXCEPT: a. Nervousness b. Bradycardia c. Insomnia d. Hypertension

b. Bradycardia

Hypotension as a result of verpamil or diltiazem administration may be treated with the use of which of the following medications? a. Amiodarone b. Calcium chloride c. Nitroglycerin d. Vasopressin

b. Calcium chloride

The pharmacological effects of administering insulin to a hyperglycemic patient is an example of increased: a. Biotransformation b. Carrier-mediated diffusion c. Passive transport d. Transference

b. Carrier-mediated diffusion

Your patient, a known alcoholic, has developed a cardiac dysrhythmia that you intend to treat with synchronized cardioversion. Because he is awake, you decide to administer diazepam to ease this experience. However, you find that to induce the desired state you must administer more than twice the normal dose of diazepam. This condition is known as: a. Cumulative effect b. Cross-tolerance c. Tachyphylaxis d. Idiosyncrasy

b. Cross-tolerance

Loop diuretics achieve their therapeutic effects by: a. Decreasing the absorption of sodium at the glomerulus b. Decreasing the reabsorption of sodium at the ascending Loop of Henle c. Enhancing reabsorption throughout the Loop of Henle d. Increasing the reabsorption of sodium at the ascending Loop of Henle.

b. Decreasing the reabsorption of sodium at the ascending Loop of Henle

Which of the following reflects signs and symptoms of lidocaine toxicity? a. Yellow vision, Blured vision, anorexia, diarrhea b. Disorientation, drowsiness, parethesia, muscle twitching c. Jaundice color, Ascites, postural hypotension d. Flushing, dyspnea, chest pain, increased myocardial oxygen demand.

b. Disorientation, drowsiness, parethesia, muscle twitching

You are caring for a middle-aged male patient who has a long history of psychiatric disorders. During your assessment you note excessive tremors and uncontrollable spasms in his head, neck and arms. This is most likely: a. Manifestations of his psychotic illness b. Extrapyramidal side effects of his antipsychotic medication c. A purposeful action designed to draw attention away from his medical history d. Drug interactions produced by antihypertensive medications and diuretics.

b. Extrapyramidal side effects of his antipsychotic medication

Which of the following is a product of the endocrine portion of the pancreas? a. Iodine b. Glucagon c. Synovial fluid d. Digestive enzymes

b. Glucagon

Zantac, Tagament, and Pepcid are all drugs that reduce the volume of gastric acid secretion and its acid content. These drugs are effective due to their antagonistic action at which receptor site? a. G-1 b. H-2 c. Alpha-2 d. Gastro-2

b. H-2

If a patient is taking Enfuvirtide, a protease inhibitor, you should be MOST suspicious that he or she has: a. cancer b. HIV infection c. tuberculosis d. a bacterial infection

b. HIV infection

When administered at an infusiton rate of 10 to 20 mcg/kg/minute, expected effects of dopamine administration include: a. Renal and mesenteric vasodilation. b. Improvement in blood pressure. c. Reduction in heart rate. d. Decreased stroke volume.

b. Improvement in blood pressure.

Morphine sulfate, when used to alleviate the pain associated with an acute myocardial infarction, should be given: a. In 10 mg IM injections every 2 to 3 hours as needed until the pain is abolished. b. In small incremental IV doses of 2 to 4 mg every 5 minutes as needed. c. In 5 to 10 mg IV doses and repeated every hour as needed. d. In 3 to 5 mg IV dose every 10 minutes as needed until he pain is abolished.

b. In small incremental IV doses of 2 to 4 mg every 5 minutes as needed.

MAO inhibitors block the enzyme monoamine oxidase thus leading to increased levels of norepinephrine. MAO inhibitors interact with a variety of other drugs. Which of the following is a list of MAO inhibitors? a. Lithium, Desyrel, and Zoloft b. Marplan, Nardil, and Parnate c. Prozac, Wellbutrin, and Zoloft d. Bupropion, Fluoxetine, and Trazodone

b. Marplan, Nardil, and Parnate

Which of the following is a proton-pump inhibitor? a. Cimetidine (Tagament) b. Omeprazole (Prilosec) c. Pirenzepine (Gastrozepine) d. Rantidine (Zantac)

b. Omeprazole (Prilosec)

Hydrocodone, propoxyphene, and oxycodone are all examples of: a. Barbiturates b. Opiod analgesics c. Sedative-Hypnotics d. Phenothiazines

b. Opiod analgesics

Which of the following statements about histamines and antihistamine is NOT true? a. A chief side effet of antihistamines is sedation b. Stimulation of the H1 histamine receptors plays a major role in the development of anaphylaxis c. Although asthma attacks can be caused by allergens, antihistamines should not be administered as a treatment d. There are antagonists for both H1 and H2 histamine receptor sites.

b. Stimulation of the H1 histamine receptors plays a major role in the development of anaphylaxis

Part of the reason that tricyclic antidepressants (TCAs) have fallen somewhat out of favor to selective serotonin reuptake inhibitors (SSRIs) is becasue: a. SSRIs have the added advantage of also blocking norepinephrine and dopamine b. TCA overdoses have cardiotoxic effect c. SSRIs have the added advantage of also blocking histamine and acetylcholine d. TCA overdoses often create a hypertensive crisis in patients.

b. TCA overdoses have cardiotoxic effect

Drug allergies may be divided into four categories based on the body's immune response. Hemolytic anemia and the destruction of platelets suggest which category of drug reaction? a. Type I b. Type II c. Anaphylactic d. Cross reactivity

b. Type II

Medical direciton may prescribe a pharmacological agent for sedation and seizure conrol in a heat stroke pation. The drug of choice for this patient will be: a. Benadryl 50mg IM b. Versed 2.0mg IM c. Scopolamine 12.5mg PO d. Morphine Sulfate 5.0mg IV

b. Versed 2.0mg IM

Most psychotherapeutic medications work by blocking ________ receptors in the brain. a. melatonin b. dopamine c. epinephrine d. norepinephrine

b. dopamine

blood pressure equation

blood pressure = cardiac output x peripheral vascular resistance

The standard subcutaneous dose of epinephrine used in treatment of the adult with severe anaphylaxis is: a. 0.1 to 0.3 mg of 1:1000 solution b. 0.1 to 0.3 mg of 10,000 solution c. 0.3 to 0.5 mg of 1:1000 solution d. 0.3 to 0.5 mg of 1:10,000 solution

c. 0.3 to 0.5 mg of 1:1000 solution

The recommended dose of aspirin for the patient experiencing the acute coronary syndrome is: a. 40 to 81 mg. b. 81 to 162 mg. c. 162 to 324 mg. d. 325 mg.

c. 162 to 324 mg. if you put, d. 325 mg, you'll only get partial credit.

You are dispatched to the home of a 47-year-old male who is having difficulty breathing. He informs you that he has a history of asthma and has been laboring to breathe for the past 2 hours. Wheezes are auscultated in all lung fields, and accessory muscle use in visible at the clavicles. Which beta-2 selective medication might you consider administering to this patient? a. Isuprel, 2ug/min b. Epinephrine 0.3mg 1:1000 SQ c. Albuterol 2.5mg d. Atrovent 0.5mg

c. Albuterol 2.5mg

Which of the following classifications of medication does NOT produce sedation? a. Barbiturates b. Opioid agonists c. Amphetamines d. Benzodiazepines

c. Amphetamines

Epinephrine is classified as an adrenergic substance and has an affinity for both alpha and beta receptor sites. Epinephrine is: a. An enzyme produced by the kidneys b. An enzyme produced in the cerebral cortex c. An hormone produced by the adrenal medulla located near the kidneys d. A hormone produced by the medulla oblongata located at the base of the brain.

c. An hormone produced by the adrenal medulla located near the kidneys

Naloxone is an _________ to opioid receptors. a. Agonist b. Agonist-antagonist c. Antagonist d. Analgesic

c. Antagonist

Medications instilled directly into the auditory canal to treat ear problems include all of the following, EXCEPT: a. Antibacterial agents b. Topical anesthetics c. Anticholinergic agents d. Ceruminolytic agents.

c. Anticholinergic agents

A concern for patients taking both insulin and beta-blockers is: a. The two medications, when taken together, have a synergistic effect b. Beta-blockers, due to their sympatholytic effects on the nervous system, can further reduce available ATP to the cells of the body in the presence of insulin c. Beta2 adrenergic blockers can both hide the effects of hypoglycemia, making it difficult to recognize, and decrease the release of glucagon d. Beta-blockers prevent the release of insulin from the beta cells in the pancreas

c. Beta2 adrenergic blockers can both hide the effects of hypoglycemia, making it difficult to recognize, and decrease the release of glucagon

The drug of choice for cardiogenic shock is: a. Epinephrine 1mg IVP d. Furosemide 20 mg Slow IVP c. Dopamine 5ug/kg/min IV drip d. Atropine 0.5mg IVP

c. Dopamine 5ug/kg/min IV drip

Which body pancreatic hormone, that is also available in a synthetic drug preparation, stimulates the liver to break down glycogen so that glucose is released into the blood? a. Insulin b. Angiotensinogen c. Glucagon d. Epinephrine

c. Glucagon

Which of the following is NOT a common side effect of a loop diuretic? a. Orthostatic hypotension b. Hypokalemia c. Hypernatremia d. Reflex tachycardia

c. Hypernatremia

A lesion in the posterior pituitary gland resulting in hypersectretion of ADH may result in: a. Hypercalcemia b. Hypokalemia c. Hypertension d. Hyponatremia

c. Hypertension

Loop diuretics are powerful, short-acting agents that: a. Inhibit sugar and water reabsorption in the loop of Henle b. Facilitate sugar and water reabsorption in the loop of Henle c. Inhibit sodium and chloride reabsorption in the Loop of Henle d. Facilitate sodium and chloride reabsorption in the loop of Henle.

c. Inhibit sodium and chloride reabsorption in the Loop of Henle

At the cellular level, the treatment of seizures is generally accomplished by __________ the influx of __________ ions into the neural cells. a. Enhancing, potassium b. Inhibiting, potassium c. Inhibiting, sodium calcium d. Enhancing, sodium and calcium

c. Inhibiting, sodium calcium

Histamine is a chemical that plays an important role in the body's response to an antigen. Which of the following is true of histamine? a. It helps to prevent dangerous bronchoconstriction b. It increases cardiac output to counteract peripheral blood pooling c. It is concentrated highest in the skin, lungs, and gastrointestinal tract d. It counteracts antihistamine chemicals by decreasing the swelling of tissues and decreasing capillary permeability

c. It is concentrated highest in the skin, lungs, and gastrointestinal tract

Which of the following would be important to re-evaluate after giving a medication for induction of sedation? a. Pupils, heart rate and blood glucose level b. Level of consciousness, respirations and blood glucose level c. Level of consciousness, respirations and blood pressure d. Pupils, heart rate and painful stimuli

c. Level of consciousness, respirations and blood pressure

Which statement about levodopa in the treatment of Parkinson's disease is NOT true? a. Levodopa is an effective therapy because it converts dopamine in the brain. b. Levodopa crosses the blood-brain barrier. c. Levodopa remains an effective therapy throughout the disease. d. Levodopa is often combined with another ingredient to prevent metabolism outside of the brain.

c. Levodopa remains an effective therapy throughout the disease.

You are dispatched to the home of a terminally ill cancer patient who is on a morphine pump. Her respiration's are 6 per minutes and shallow. His family thinks he may have overdosed on the morphine. Which opiod antagonist medication might you administer to this patient? a. Romazacon 1.0 mg d. Dextrose 25g c. Naloxone 2.0mg d. Thiamine 100mg

c. Naloxone 2.0mg

One important distinction with potassium-sparing diuretics over other forms of diuretics is that potassium-sparing diuretics: a. Exert their effects early in the nephritic process b. Are more efficient than loop diuretics in sodium and water reabsorption c. Not only decrease sodium reabsorption, they increase potassium reabsorption. d. Are so potent that they are rarely used with other diuretics.

c. Not only decrease sodium reabsorption, they increase potassium reabsorption.

the acronym SLUDGE helps identify the effects of _______ drugs on the autonomic nervous system. a. Sympathomimetic b. Sympatholytic c. Parasympathomimetic d. Parasympatholytic

c. Parasympathomimetic

Lasix is a common loop diuretic. Patient taking diuretics may experience a significant loss of electrolytes that may lead to cardiac abnormalities and other complications. The electrolytes deficiency commonly associated with Lasix Administration is: a. Zinc b. Calcium c. Potassium d. Magnesium

c. Potassium

Diazepam has which of the following significant side effects? a. Hypoglycemia b. Thrombophlebitis c. Respiratory Depression d. Anaphlyaxis

c. Respiratory Depression

The crunching sound that is made when unlubricated skeletal parts rub against each other is called: a pleural friction rub. a synovial rub. bursal friction. crepitus.

crepitus.

With regard to the endocrine system, which of the following statements is NOT true? a. The pituitary gland is often called the "master gland." b. The posterior pituitary hormones are actually synthesized in the hypothalamus c. The hypothalamus and pituitary glands have no physical link d. The hypothalamus controls many of the function of the pituitary gland.

c. The hypothalamus and pituitary glands have no physical link

Which of the following statements about vitamins is NOT true? a. Vitamin D is unique in that it can be produced in the skin by sunlight exposure. b. The liver stores the fat-soluble vitamins; consequently, they will become deficient only after long periods of inadequate vitamin intake. c. The individual B vitamins are named for the order in which they are in demand by the body d. The water-soluble vitamins must be routinely ingested, because the body does not store them.

c. The individual B vitamins are named for the order in which they are in demand by the body

Barbiturates, once commonly prescribed for sedation, are divided into four categories based on: a. Their addictiveness b. Their therapeutic index c. Their duration of action d. Their rout e of administration

c. Their duration of action

Some drugs treat depression by increasing levels of norepinephrine and serotonin. One example of a popular antidepressant is Elavil. This drug is classified as a: a. MAO inhibitor b. Sympathetic blocker c. Tricyclic antidepressant d. Parasympathetic blocker

c. Tricyclic antidepressant

A 23-year-old female patient is complaining of lower abdominal and genital pain. Upon examination you find a white, curdlike vaginal discharge. This is most typical of: Gardnerella. candidiasis. gonorrhea. herpes simplex.

candidiasis.

A patient experiencing a cough and fever is taking isoniazid. You should suspect that he or she has: a. HIV b. malaria c. tuberculosis d. pneumonia

c. tuberculosis

cardiac output equation

cardiac output = heart rate x stroke volume

To assess an infant's airway, you should: perform a jaw-thrust maneuver. carefully extend the head and neck. hyperextend the head and lift the chin. perform a tongue-jaw lift.

carefully extend the head and neck.

pathogenic

caused by viruses, bacteria, and other types of germs, in reference to a disease

most frequently occurring anions

chloride (Cl-), bicarbonate, phosphate

During the cardiac cycle, the S2 heart sound indicates the ________ of the ________ valves. closing; aortic and pulmonic opening; mitral and tricuspid closing; mitral and tricuspid opening; aortic and pulmonic

closing; aortic and pulmonic

Nail condition cause by chronic hypoxia associated with COPD

clubbing

A patient experienced a brief loss of consciousness following a blow to the head. During your questioning, she describes a series of events that do not match what observers had noted before the trauma. This might be an example of: confabulation. incoherence. delusion. mania.

confabulation.

A complete neurological exam includes assessment of mental status and speech, the motor and sensory systems, reflexes, and the:

cranial nerves

A complete neurologic exam includes assessment of mental status and speech, the motor and sensory systems, reflexes, and the: speed of peripheral nervous conduction. cranial nerves. visual acuity. tympanic membrane.

cranial nerves.

Areas of a neurologic exam include all of the following EXCEPT: reflexes. cranium. cranial nerves. mental status and speech.

cranium.

What is the initial and repeat dosing schedule for sodium bicarbonate administration? a. 1 ampule; the repeat dose is 1/2 ampule every 10 minutes b. 1 mEq/kg; the repeat dose is 1 mEq every 10 minutes c. 1 mg/kg; the repeat dose is 1/2 of the initial dose every 10 minutes d. 1 mEq/kg; the repeat dose is 1/2 the initial dose every 10 minutes

d. 1 mEq/kg; the repeat dose is 1/2 the initial dose every 10 minutes

When lidocaine is used int he management of ventricular fibrillation, the recommended dose is: a. 6 mg rapid IV bolus repeated if necessary in 1 to 2 minutes with 12 mg. b. 1 to 3 mg titrated to effect. c. 0.5 to 1.0 mg/kg slow IV bolus d. 1 to 1.5mg/kg repeated in 3 to 5 minutes to maximum dose of 3 mg/kg.

d. 1 to 1.5mg/kg repeated in 3 to 5 minutes to maximum dose of 3 mg/kg.

You are instructed to administer 10% dextrose to you newborn patient. You drug box contains an ampule of 50% dextrose. You will be able to prepare a solution of 10% dextrose by diluting in the ratio of glucose:saline a. 1:1 b. 1:2 c. 1:3 d. 1:4

d. 1:4

Amiodarone is: a. An anticholinergic medication used in the management of symptomatic bradycardia medication used int he management of symptomatic bradycarida. b. A beta blocker used for short-term management of congestive heart failure. c. A arrhythmic medication used int he suppression of Asystole. d. An Antiarrythmic useful in the management of both atrial and ventricular tachydysrhythmias.

d. An Antiarrythmic useful in the management of both atrial and ventricular tachydysrhythmias.

In parasympathetic nervous action, the binding of acetylcholine at nicotinic receptors produces: a. A blocking of the effector organ b. A relaxation of the effector organ c. An inhibition of the effector organ d. An excitatory response in the effector organ

d. An excitatory response in the effector organ

Medications such as lorazepam (Ativan) and diazepam (valium): a. Are schedule II drugs and have a very high abuse and addiction potential b. Are schedule III drugs that are opioids but have a low potential for dependence. c. Are schedule V drugs and have the lowest abuse potential of all controlled substances d. Are schedule IV drugs that have a low abuse potential and limited dependence potential

d. Are schedule IV drugs that have a low abuse potential and limited dependence potential

Patients with COPD and in particular emphysema usually responds best to which of the following pharmacological agents: a. Albuterol 2.5 mg b. Epinephrine 0.5mg 1:1000 SQ c. Diphenhydramine 25mg IM d. Atrovent 0.5 mg

d. Atrovent 0.5 mg

In the treatment of acute glaucoma, which classification of drugs are most useful? a. Alpha agonists and anticholinergics b. Beta agonists and anticholinergics c. Anticholinergics and beta blockers d. Beta-blockers and cholinergics

d. Beta-blockers and cholinergics

With changes in dietary habits, health, and activity, many geriatric patients rely on laxatives to avoid constipation. Abuse of laxatives may result in: a. Hemorrhoids b. Eating disorders c. Diabetes (Type II, adult on set) d. Bowel damage and electrolyte imbalance

d. Bowel damage and electrolyte imbalance

Opium and its derivatives are categorized as: a. Barbiturates b. Synthetic stimulants c. Mineral-based drugs d. Controlled substances

d. Controlled substances

What is the drug of choice when treating patients having generalized motor (grand mal) seizures a. Lithium b. Romazicon c. Valproic Acid d. Dilantin

d. Dilantin

Your patient is a 65 year old female complaining of dyspnea with audible wheezes and rales. Vital signs reveal: P 140, BP 160/110, R 24. Peripheral edema is noted as well. What class of medications could benefit this patient. a. Calcium channel blocker b. Sympathomimetics c. Parasymptholytics d. Diuretics

d. Diuretics

What drug administration route is subject to the "first pass effect?" a. Pulmonary b. Percutaneous c. Parenteral d. Enteral

d. Enteral

The treatment of choice for an acute anaphylactic reaction is to administer: a. Epinephrine 0.3mg 1:1000 IVP b. Solu-Medrol 125mg IVP c. Benadryl 25mg IM d. Epinephrine 0.3-0.5mg 1:1000 SQ

d. Epinephrine 0.3-0.5mg 1:1000 SQ

the treatment of choice for an acute anaphylactic reation is to administer: a. Epinephrine 0.3mg 1:1000 IVP b. Solu-Medrol 125mg IVP c. Benadryl 25mg IM d. Epinephrine 0.3mg 1:1000 SQ

d. Epinephrine 0.3mg 1:1000 SQ

A diuretic that affects reabsorption early in the nephron filtration process will: a. Affect water reabsorption but not electrolytes b. Affect electrolytes but not fluid reabsorption c. Have a lesser impact of water an sodium removal d. Have a greater impact of water and sodium removal.

d. Have a greater impact of water and sodium removal.

What is the assigned schedule number of phenobarbital according to the Controlled Substance Act of 1970? a. I b. II c. III d. IV

d. IV

All of the following statements about nonsteroidal anti-inflammatory drugs (NSAIDs) are TRUE EXCEPT that NSAIDs: a. Are used as analgesics and antipyretics b. Are prescribed to relieve pain following trauma and surgery c. Interfere with the production of prostaglandins, thereby interrupting the inflammatory process d. Include aspirin, acetaminophen and ibuprofen.

d. Include aspirin, acetaminophen and ibuprofen

All laxatives are similar, in that they: a. Reduce the molecular bonds of colon contents b. Increase peristalsis via direct stimulation c. Increase bulk in the intestines d. Increase the water content in the colon

d. Increase the water content in the colon

Medications to treat extrapyramidal side effects from antipsychotic medications, as well as drugs used to treat Parkinson's disease, achieve their therapeutic effects by: a. Enhancing the effects of acetylcholine, an inhibitory neurotransmitter, in the basal ganglia b. Blocking the release of dopamine from the substantia nigra in the basal ganglia c. Blocking the transmission of neural impulses through the pyramidal decussation in the brain. d. Increasing the amount of dopamine at the receptor sites in the basal ganglia.

d. Increasing the amount of dopamine at the receptor sites in the basal ganglia.

When administering glucagon to a patient, it is important to remember that: a. Glucagon can only be administered intravenously b. It is only effective in patients with elevated blood glucose levels c. Glucagon will stimulate glycogeneisis in a hypoglycemic patient d. It increases the rate of gluconeogensis

d. It increases the rate of gluconeogensis

Which of the following is NOT a characteristics of nifedipine (Procardia)? a. It has little effect on preload, so orthostatic hypotension is not a problem. b. Reduction of blood pressure is accomplished by reducing the peripheral vascular resistance. c. It effectively dilates the coronary arteries. d. It produces a positive inotropic effect of the heart.

d. It produces a positive inotropic effect of the heart.

Benzodiazepines are among the most widely prescribed drugs. Their safe therapeutic index ratio makes them a popular drug to prescribe for anxiety-related symptoms. The actions of this class of drug include: a. Euphoria, anesthesia relaxation, and anxiety reduciton b. Anxiety reduction, mild hallucinogen, and mild hypnotic c. Hypnosis, euphoria, anxiety rexuction and mild hallucinogen d. Muscle relaxation, anxiety reduction, sedative and anticonvulsant

d. Muscle relaxation, anxiety reduction, sedative and anticonvulsant

Two drugs that augment or replace posterior pituitary hormones are: a. Insulin and glucagon b. Glucagon and vasopressin c. Adenosine and vasopessin d. Oxycontin and antidiuretic hormone

d. Oxycontin and antidiuretic hormone

The principal mechanism of action of Dilantin in treating status seizures is the: a. Blocking of GABA receptor sites to prevent further cellular activity in the CNS b. Influx of calcium ions through calcium channels to enhance depolarization c. Blocking of acetylcholine receptor sites a the neuromuscular junction to prevent convulsions d. Prevention of sodium ions entering the cell by closing sodium channels.

d. Prevention of sodium ions entering the cell by closing sodium channels.

Magnesium sulfate is indicated in the treatment of: a. Asystole b. Paroxysmal supraventricular tachycarida c. Hemodynamically significant Bardydysrhythmias d. Pulseless ventricular tachycardia/ventricular fibrillation.

d. Pulseless ventricular tachycardia/ventricular fibrillation.

Which of the following drugs is effective in reversing benzodiazepine induced sedation and coma? a. Centrax b. Librium c. Dalmane d. Romazicon

d. Romazicon

Prolonged deficiency of vitamin C may result in: a. Pernicious anemia b. Increased bleeding c. Kwashiorkor d. Scurvy

d. Scurvy

You are dispatched to a local mental health facitlity for a 42-year-old female who is acting inappropriately. upon arrival, you find the patient sitting in a chair in no apparent physical distress. She continually blinks her eyes (excessively) and is smacking her lips. She informs you that she has schizophrenia and has been taking Thorazine for many years. Her presentation may be attributed to: a. Ocular Toxemia b. Senile Dementia c. Bipolar disorder d. Tardive dyskinesia

d. Tardive dyskinesia

What medication can you use to prevent exacerbation of Wemicke-Korsakoff Syndrome that commonly affects the chronic alcoholic? a. Dextrose 50% 25G IVP b. Narcan 2.0mg IVP c. Glucagon 1 mg IM d. Thiamine 100mg IM

d. Thiamine 100mg IM

Stimulation of beta1 adrenergic receptors would produce all of the following effects, EXCEPT: a. an increase in heart rate. b. enhanced cardiac automaticity c. increased cardiac electrical conduction d. decreased cardiac contractility

d. decreased cardiac contractility

Normal pediatric respiratory and heart rates ________ as children grow older. stay the same decrease increase increase, then decrease

decrease

Cyanosis is caused by

deoxyhemoglobin

While speaking, your patient shifts rapidly from topic to topic without realizing that his thoughts are not connected. This might be a sign of any of the following conditions EXCEPT: psychosis. schizophrenia. depression. manic episodes.

depression.

You are examining a 70-year-old female patient with altered mental status. She is awake and can speak. You detect an odor of acetone and suspect that the patient is suffering from: bowel obstruction. alcohol poisoning. diabetic ketoacidosis. cyanide poisoning.

diabetic ketoacidosis.

The mitral valve is open during: repolarization. cardiac standstill. diastole. systole.

diastole.

Palpation and inspection of the point of maximal impulse can reveal certain conditions, such as: cardiac tamponade. coronary insufficiency. enlarged right ventricle. pneumothorax.

enlarged right ventricle.

the most common type of blood cell

erythrocytes

An ECG monitor is useful for: evaluating the effectiveness of cardiac contractions. evaluating the heart's electrical conduction system for abnormalities. determining cardiac output. determining stroke volume.

evaluating the heart's electrical conduction system for abnormalities.

You can test the oculomotor, trochlear, and abducens nerves by: conducting a visual acuity test. evaluating the patient's extraoccular movements. shining a light into the pupils and observing constriction of the pupils. closing one nostril while presenting a strong smell under the other nostril

evaluating the patient's extraocular movements.

During the reassessment of a stable patient, you should recheck vital signs every 15 minutes. This is an example of: reflecting. evaluating. reviewing. applying principles.

evaluating.

Capnometry measures the partial pressure of CO2 in: inspired air. venous blood. arterial blood. expired air.

expired air.

Bronchophony can reveal

fluid in the lungs

When evaluating dorsiflexion, you should instruct the patient to point his: hand upward. foot downward. foot upward. hand downward.

foot upward.

covalent bond

force holding atoms together that results when atoms share electrons. the strongest of the three types of chemical bonds

The first step in critical thinking is to: determine statistically significant conditions. ascertain the chief complaint. conduct a focused exam. form a concept.

form a concept.

As you survey the scene, you are gathering information about your patient and the scene. This step in the critical decision-making process is called: interpreting the data. reflecting on the incident. evaluating results. forming a concept.

forming a concept.

A patient who is experiencing dysphonia: has voice changes secondary to vocal cord problems. has defective speech caused by motor deficits. has defective language caused by neurological damage. is speaking clearly and normally.

has voice changes secondary to vocal cord problems.

The precordial leads provide a view of the: horizontal plane of the heart, including the left ventricle and septum. inferior portion of the heart, including the entire right ventricle. horizontal plane of the heart, including the right ventricle and right atrium. lateral plane of the heart, including part of the right ventricle, most of the left ventricle, and the septum.

horizontal plane of the heart, including the left ventricle and septum.

During a neurologic exam, you ask your patient to stick out his tongue. It deviates to the right. This may indicate a lesion to the patient's ________ nerve. vagus glossopharyngeal facial hypoglossal

hypoglossal

shock

hypoperfusion; inadequate perfusion of the body tissues, resulting in an inadequate supply of oxygen and nutrients to the body tissues

A patient who is misinterpreting what is happening is experiencing a(n): delusion. illusion. hallucination. paresthesia.

illusion.

shock signs/symptoms: neurogenic

injury to brain or spinal cord, warm and dry red skin, low blood pressure and slow pulse, hypotension, reflex bradycardia

A 12-lead ECG that reveals ST elevation in all of the precordial leads most indicates myocardial: injury to the right ventricle and septum. infarction to the left and right ventricles. ischemia to the left ventricle and septum. injury to the right ventricle, left ventricle, and septum.

injury to the right ventricle, left ventricle, and septum.

You respond to a patient complaining of lower abdominal pain. Upon your arrival, you find a 25-year-old man who also complains of groin pain. After inspecting the penis and scrotum, your next step would be to: lift the scrotum and inspect for swelling or lumps. take a urine sample. inspect the glans. palpate the testicles.

inspect the glans.

The standard sequence for examining the chest is: inspect, palpate, auscultate, percuss. inspect, palpate, percuss, auscultate. palpate, percuss, auscultate, inspect. inspect, auscultate, percuss, palpate.

inspect, palpate, percuss, auscultate.

After considering the information you have collected, you determine the most likely cause of your patient's condition. This step in the critical decision-making process is called: forming a concept. interpreting the data. applying the principles. reflecting on the incident.

interpreting the data.

the compartment that has the most body water

intracellular (70%)

Using knowledge and experience to diagnose patients and plan their treatment is called clinical: intuition. judgment. practice. reflection.

judgment.

During the abdominal exam, areas that are known to be painful or tender should be examined: last. first. only by a physician. at the beginning and end of the exam.

last.

Cullen sign is a ___________ indicator of intrabdominal bleeding

late

To inspect and examine the anus, a paramedic should place the patient in the ________ position. kneeling prone left lateral recumbent supine

left lateral recumbent

A condition marked by exaggerated lumbar concavity is called: lordosis. spondylosis. kyphosis. scoliosis.

lordosis.

When evaluating any patient's genitalia, a paramedic must always: maintain privacy. use a lubricant. be of the same gender as the patient. perform a rectal exam, as well.

maintain privacy.

You are evaluating a patient who was a restrained driver of a motor vehicle that crashed into the median barrier at 40 mph. He is awake, alert, and oriented, complaining of neck and back pain. There is a bruise across his upper abdomen in the shape of the seat belt. Based on this information, you should assume he is a ________ trauma patient, and transport him to ________. minor; the closest hospital major; the closest hospital minor; the patient's hospital of choice major; a trauma center

major; a trauma center

You are sent to the home of an insulin-dependent diabetic female patient. You ask her to state her name, the month, and her address. This would be an example of assessing the patient's: judgment. general fund of knowledge. mood. memory and attention.

memory and attention.

aerobic

metabolism requiring oxygen

anaerobic

metabolism without oxygen

When oxygen is not bound to the iron molecule of hemoglobin, the iron molecule is in the ferrous (Fe2+) charge state, rather than when oxygen changes the charge of iron to the ferric (Fe3+) state. A dangerous form of hemoglobin that is unable to oxidize is: oxyhemoglobin (OxHb). carboxyhemoglobin (COxHb). methemoglobin (MetHb). deoxyhemoglobin (DeoHb).

methemoglobin (MetHb).

active transport

movement of a substance through a cell membrane against the osmotic gradient; that is, from an area of lesser concentration to an area of greater concentration, opposite to the normal direction of diffusion; requires the use of energy to move a substance

osmosis

movement of solvent (water) in a solution from an area of lower solute concentration to an area of higher solute concentration

When responding to calls involving two-vehicle collisions, you must always suspect: flail chest. multiple patients. severe lower extremity injuries. patients with altered mental status.

multiple patients.

An elevation of the ST segment is associated with: slowed conduction through the AV node. hyperkalemia. myocardial injury. left ventricular hypertrophy.

myocardial injury.

The presence of inverted T waves on an ECG indicates: myocardial instability. myocardial necrosis. myocardial ischemia. myocardial infarction.

myocardial ischemia.

A 45-year-old patient presents with abdominal pain in the right lower quadrant. Upon auscultation of bowel sounds, you hear high-pitched gurgles and clicks lasting around 10 seconds. This patient may have: borborygmi. paralytic ileus. hyperperistalsis. normal sounds.

normal sounds.

You arrive at a location and begin to read the scene by: observing the immediate surroundings. addressing life threats. approaching the patient. conducting a focused exam.

observing the immediate surroundings.

idiopathic

of unknown cause, in reference to a disease

To detect life-threatening cardiac dysrhythmias, the paramedic must view the ECG in ________ lead(s). two one twelve three

one

Your 23-year-old female patient presents with relatively normal vital signs and is fully alert and oriented. Her only complaint is lower abdominal pain. If you assume she is pregnant, you are:

overgeneralizing

Your 23-year-old female patient presents with relatively normal vital signs and is fully alert and oriented. Her only complaint is lower abdominal pain. If you assume she is pregnant, you are: overgeneralizing. evaluating. interpreting the data. applying principles.

overgeneralizing.

pH of acidosis in the body

pH < 7.35

pH of alkalosis in the body

pH > 7.45

Components of a scene size-up include all of the following EXCEPT: mechanism of injury. dangers to bystanders. patient's chief complaint. number of patients.

patient's chief complaint.

a likely cause of hypovolemic shock

penetrating trauma

Paramedics treat patients with the same techniques as other clinicians, EXCEPT that they: are not allowed to make independent decisions. perform these procedures in uncontrollable and unpredictable environments. are far less trained than other practitioners. treat life-threatening emergencies only.

perform these procedures in uncontrollable and unpredictable environments.

During your focused physical exam of a patient with a suspected myocardial infarction, you should examine: extraocular movements. peripheral perfusion status. deep tendon reflexes. range of motion in joints.

peripheral perfusion status.

Rebound tenderness in the abdomen is a sign of potential: kidney stone. food poisoning. peritoneal irritation. full urinary bladder.

peritoneal irritation.

Devices for maintaining airway patency in a five-year-old patient include all of the following EXCEPT a(n): suction unit. oropharyngeal airway. endotracheal tube. pharyngotracheal lumen airway.

pharyngotracheal lumen airway.

While you are assessing for Babinski's response, the patient's big toe dorsiflexes and the other toes fan out. This indicates a ________ response, which is ________. negative; normal positive; abnormal positive; normal negative; abnormal

positive; abnormal

the most abundant intracellular cation

potassium (K+)

congenital

present from birth, in reference to a disease

facilitated diffusion

process in which carrier proteins transport large molecules across the cell membrane (ex: glucose entering a body cell)

Sign of gonorrhea in men

profuse, yellow discharge

You are working at the triage desk in the ED when a young man on crutches approaches the desk. He appears moderately short of breath. He states he had a cast put on his left leg seven days ago after surgery for a ruptured Achilles tendon. This morning, while he was watching television, he suddenly became short of breath. He has a history of asthma, for which he takes Xopenex as needed, and is taking Tylenol with codeine for pain related to his surgery. The patient's lung sounds are clear and equal, SpO2 is 90 percent on room air, heart rate is 100 and regular, respirations are 24 and slightly labored, blood pressure is 128/88 mmHg, and the patient is afebrile. These findings are most consistent with: asthma exacerbated by recent anesthesia. pulmonary embolism associated with immobilization of the lower extremity. pneumonia secondary to recent anesthesia. allergic reaction to codeine.

pulmonary embolism associated with immobilization of the lower extremity.

Blood entering the left atrium arrives via the: superior and inferior vena cava. pulmonary vein. bicuspid valve. pulmonary artery.

pulmonary vein.

A fast, systematic assessment designed to identify other life-threatening injuries after the primary assessment is the: detailed physical exam. focused physical exam. 1-minute exam. rapid secondary assessment.

rapid secondary assessment.

Asking your patient to recall what he had for a meal earlier in the day would test: recent memory. remote memory. long-term memory. immediate memory.

recent memory.

After a call you discuss the patient with the ED physician, comparing diagnoses and care plans. This is an example of: reflecting. evaluating. applying principles. putting it all together.

reflecting.

cause of metabolic alkalosis

relatively uncommon and due to an increase in bicarbonate (HCO-3) levels or a decrease in circulating acids. results from an abnormal loss of hydrogen ions (H+), an increase in HCO-3 levels, or a decrease in extracellular fluid levels. vomiting is the most common cause.

metaplasia

replacement of one type of cell by another type of cell that is not normal for that tissue

Your three-year-old patient opens her eyes and responds when you speak to her. Her mental status is best described as: responsive to tactile stimuli. responsive to verbal stimuli. alert. lethargic.

responsive to verbal stimuli.

Changing or stopping interventions that are NOT working is an example of: revising the management plan. reading the patient. reevaluating. reviewing your performance.

revising the management plan.

Jugular venous pressure approximates the patient's ________ pressure. right atrial left ventricular right ventricular left atrial

right atrial

The SA and AV nodes are perfused by the ________ artery. anterior descending circumflex coronary right coronary left coronary

right coronary

shock signs/symptoms: anaphylactic

s/s often appear within a minute or less, flushing, itching, hives, swelling, cyanosis, laryngeal edema, wheezing, stridor, breathing difficulty, vasodilation, increased heart rate, decreased blood pressure, nausea, vomiting, diarrhea, altered mental status, dizziness, seizures, headache

Heart sound S2 may be heard at the end of systole by auscultating over the ________ intercostal space. first second sixth fifth

second

The careful, thorough process of eliciting a patient's history and conducting a physical exam is known as the: rapid medical assessment. primary survey. secondary assessment. ongoing assessment.

secondary assessment.

shock signs/symptoms: cardiogenic

severe left ventricle failure secondary to acute myocardial infarction or congestive heart failure, pulmonary edema, diminished lung sounds, productive cough, cyanosis, altered mentation

To assess JVD, have patient

sit at 45 degree angle

For you to assess for jugular venous distention, your patient should be ________ at a ________-degree angle. sitting; 90 sitting; 45 supine; 45 standing; 90

sitting; 45

the most abundant extracellular cation

sodium (Na+)

most frequently occurring cations

sodium (Na+), potassium (K+), calcium (Ca++), magnesium (Mg++)

hypertonic

state in which a solution has a higher solute concentration on one side of a semipermeable membrane that on the other side; having a greater concentration of solute molecules

hypotonic

state in which a solution has a lower solute concentration on one side of a semipermeable membrane than on the other side; having a lesser concentration of solute molecules

isotonic

state in which solutions on opposite sides of a semipermeable membrane are in equal concentration; equal in concentration of solute molecules

Trending vital signs is best accomplished by: comparing the prehospital vital signs to the patient's normal vital signs. estimating any values that are missing on the prehospital care record. taking multiple sets of vital signs. having each set of vital signs verified by a second EMS provider.

taking multiple sets of vital signs.

On a properly calibrated ECG machine, 1 mV is equivalent to a height of ________ boxes. five small one large four large ten small

ten small

pH

the abbreviation for potential of hydrogen

preload

the amount of blood delivered to the heart during diastole (when the heart fills with blood between contractions); in cardiac physiology, defined as the tension of cardiac muscle fiber at the end of diastole

the mechanism that buffers the acidity of the blood the quickest

the carbonic acid-bicarbonate buffer system

To avoid the carotid sinus, you should palpate for a carotid pulse at the level of: Terry's point. the cricoid cartilage. the sternocleidomastoid muscle. carotid bifurcation.

the cricoid cartilage.

Patterns of deterioration in patient condition may include all of the following EXCEPT: the level of consciousness increases. skin becomes cool, pale, and moist. the respiratory rate significantly increases or decreases. the heart rate significantly increases or decreases.

the level of consciousness increases.

the three elements necessary for adequate perfusion

the pump, the fluid, the container

afterload

the resistance a contraction of the heart must overcome in order to eject blood; in cardiac physiology, defined as the tension of cardiac muscle during systole (contraction)

cause of respiratory acidosis

the respiratory system cannot effectively eliminate all the carbon dioxide generated through metabolic activities in the peripheral tissues

cause of respiratory alkalosis

the respiratory system eliminates too much carbon dioxide through hyperventilation

pathophysiology

the study of the functional changes that occur within living cells and tissues that are associated with or that result from disease or injury

The ability to critically evaluate a patient's condition and formulate a treatment plan rests on all of the following EXCEPT: strictly following protocols. the ability to focus on large amounts of data. identifying and dealing with medical ambiguity. an excellent working knowledge of anatomy and physiology.

trictly following protocols.

The valve between the right atrium and right ventricle is the ________ valve. aortic tricuspid pulmonic bicuspid

tricuspid

During ventricular systole, the ventricles contract while the: tricuspid and mitral valves open. pulmonic and aortic semilunar valves close. tricuspid and mitral valves close. foramen ovale opens.

tricuspid and mitral valves close.

In vivo is a descriptive term for processes that are carried out within a living body.

true

universal blood recipient

type AB positive

universal blood donor

type O negative

To test an extremity for pain sensation, you should: briskly tap the tendon of the elbow or knee. firmly pinch a fold of skin over the most sensitive part of the extremity. use a sharp object, and have the patient tell you if he feels a sharp or dull sensation. stroke the extremity in a distal-to-proximal direction.

use a sharp object, and have the patient tell you if he feels a sharp or dull sensation.

Slowing of the electrical impulse at the AV node is necessary to allow time for the ________ to fill with blood. coronary sinus coronary arteries atria ventricles

ventricles

The QRS complex represents: ventricular repolarization. ventricular depolarization. atrial repolarization. atrial depolarization.

ventricular depolarization.

35) What is the primary constituent of plasma? A) Clotting factors B) Water C) Proteins D) Electrolytes Answer: B

water

plasma is primarily composed of

water


Related study sets

Chapter 27: The Child with a Condition of the Blood, Blood-Forming Organs, or Lymphatic System

View Set

Family Health Assessment 1 PREP U (chapters 11,12,13.)

View Set

Assistive Technology: Seating and Mobility

View Set